Skip to main content

【Pathology question】General Pathology part 1

【Pathology question】General Pathology part 1

DIRECTIONS: Each item below contains a question or incomplete statement followed by suggested responses. Select the one best response to each question.

1. Increased lipolysis of fat stores, which can result from starvation, diabetes mellitus, or corticosteroid use, is most likely to cause steatosis (fatty liver) through which one of the listed mechanisms?

    a. Decreased free fatty acid excretion from the liver leads to free fatty acid accumulation in                         hepatocytes

    b. Excess NADH (high NADH/NAD ratio) causes excess production of lactate from pyruvate, which         accumulates in hepatocytes

    c. Increased free fatty acid delivery to the liver leads to triglyceride accumulation in hepatocytes

    d. Inhibition of apoprotein synthesis by the liver leads to phospholipid accumulation in hepatocytes

    e. Inhibition of HMG-CoA reductase activity leads to cholesterol accumulation in hepatocytes

    

ANS: C

Note:

A. Increased free fatty acid excretion...

B. Excess NADH prevent oxidation of lactate from pyruvate

D. Accumulation of triglycerides in the liver.

E. Inhibition of HMG-CoA reductase activity→ indirectly increases liver LDL receptors→a reduction in cholesterol levels

Free fatty acids are normally 
taken up by the liver 
→ esterified to triglyceride 
→ converted to cholesterol 
→ oxidized into ketone bodies / incorporated into phospholipids that can be excreted from the liver as very-low-density lipoproteins (VLDLs). 

Abnormalities: Lead to the accumulation of triglycerides (fatty change or steatosis) within the hepatocytes. 

Produce hepatic steatosis include diseases that cause excess delivery of free fatty acids to the liver or diseases that cause impaired lipoprotein synthesis. 

Excess delivery of free fatty acids occurs in conditions that increase lipolysis of adipose tissue, such as starvation, diabetes mellitus, and corticosteroid use. 

Increased formation of triglycerides can result from alcohol use, as alcohol causes excess NADH formation (high NADH/NAD ratio), increases fatty acid synthesis, and decreases fatty acid oxidation. 

Impaired apoprotein synthesis occurs with carbon tetrachloride poisoning, phosphorous poisoning, and protein malnutrition.

Inhibition of HMG-CoA reductase activity is the mechanism of lovastatin, which indirectly increases liver LDL receptors and increases LDL clearance from the blood. 


2. An adult patient presents with the sudden onset of massive diarrhea. Grossly, this individual’s stool has the appearance of “rice-water” because of the presence of flecks of mucus. Cultures of this patient’s stool grow Vibrio cholerae, a curved, gram-negative rod that secretes an enterotoxin consisting of a toxic A subunit and a binding B subunit. The cholera enterotoxin causes massive diarrhea by

    a. Inhibiting the conversion of Gi-GDP to Gi-GTP

    b. Inhibiting the conversion of Gs-GTP to Gs-GDP

    c. Stimulating the conversion of Gi-GDP to Gi-GTP

    d. Stimulating the conversion of Gs-GDP to Gs-GTP

    e. Stimulating the conversion of Gs-GTP to Gs-GDP


ANS: B

Note:

Cholera toxin (霍乱毒素) & pertussis toxin (百日咳毒素) 
---altering adenyl cyclase pathway. 
---prolong the functioning of adenyl cyclase and therefore increase intracellular cAMP,
Cholera toxin inhibits the conversion of Gs-GTP to Gs-GDP
Pertussis toxin inhibits the activation of Gi-GDP to Gi-GTP. 

Second messenger systems:

  • bind to receptors that are located either on the surface of the cell or within the cell itself. 
    • Substances that react with intracellular receptors are lipid-soluble (lipophilic) molecules that can pass through the lipid plasma membrane. 
      • eg. thyroid hormones, steroid hormones, and the fat-soluble vitamins A and D. 
    • Once inside the cell these substances generally travel to the nucleus and bind to the hormone response element (HRE) of DNA.
  • React with cell surface receptors bind to guanine nucleotide regulatory proteins (G proteins)
    • Gs, Gi: Regulate cAMP (intracellular). Gs= stimulatory Gi= inhibitory
      • Activation of receptor: GDP to GTP
    • Gt: Regulate cGMP (intracytoplasmic)
    • Gq: Regulate calcium ions---IP3 pathway (intracytoplasmic)


3. In an evaluation of an 8-year-old boy who has had recurrent infections since the first year of life, findings include enlargement of the liver and spleen, lymph node inflammation, and a superficial dermatitis resembling eczema. Microscopic examination of a series of peripheral blood smears taken during the course of a staphylococcal infection indicates that the bactericidal capacity of the boy’s neutrophils is impaired or absent. What is the most likely cause of this child’s illness?

    a. Defect in the enzyme NADPH oxidase

    b. Defect in the enzyme adenosine deaminase (ADA)

    c. Defect in the IL-2 receptor

    d. Developmental defect at the pre-B stage

    e. Developmental failure of pharyngeal pouches 3 and 4


ANS: A

Note:

B. Autosomal recessive (Swiss) form of severe combined immunodeficiency disease (SCID)

C. X-linked recessive form of SCID

D. X-linked agammaglobulinemia of Bruton

E. DiGeorge’s syndrome


Chronic granulomatous disease 
  • Definition: Defective functioning of phagocytic neutrophils and monocytes due to an inability to produce hydrogen peroxide. (decreased oxidative or respiratory burst)
  • Cause: Defective NADPH oxidase, 
    • which is an enzyme on the membrane of lysosomes that converts O2 to superoxide and stimulates oxygen burst. 
    • This deficiency results in recurrent infections with catalasepositive organisms, such as S. aureus. 
  • The classic form: boys and causes death before the age of 10. 
  • Key findings: lymphadenitis, hepatosplenomegaly, eczematoid dermatitis, pulmonary infiltrates that are associated with hypergammaglobulinemia, and defective ability of neutrophils to kill bacteria. 


4. A 24-year-old female presents with severe pain during menses (dysmenorrhea). To treat her symptoms, you advise her to take indomethacin in the hopes that it will reduce her pain by interfering with the production of

    a. Bradykinin

    b. Histamine

    c. Leukotrienes

    d. Phospholipase A2

    e. Prostaglandin F2


ANS: E

Note:

Inhibit metabolic of arachidonic acid → control of acute inflammation

Corticosteroids 

  • induce the synthesis of lipocortins, a family of proteins that are inhibitors of phospholipase A2
  • decrease the formation of arachidonic acid and its metabolites, prostaglandins and leukotrienes. 

Aspirin, indomethacin, and other nonsteroidal anti-inflammatory drugs (NSAIDs),

  • inhibit cyclooxygenase
  • therefore inhibit the synthesis of prostaglandins and thromboxanes. 


Prostaglandin E2 (PGE2):

  • Fever: interleukin 1 secretion from leukocytes 
    • aspirin: inhibiting PGE2 production. 
  • Vasodilator that can keep a ductus arteriosus open. 
    • At birth, breathing decreases pulmonary resistance and reverses the flow of blood through the ductus arteriosus. The oxygenated blood flowing from the aorta into the ductus inhibits prostaglandin production and closes the ductus arteriosus. Therefore prostaglandin E2 can be given clinically to keep the ductus arteriosus open, while indomethacin can be used to close a patent ductus. 

Prostaglandin F2 (PGF2) 

  • Dysmenorrhea: uterine contractions
    • Indomethacin: inhibiting the production of PGF2.

Bradykinin

  • increases vascular permeability
  • contracts smooth muscle
  • dilates blood vessels
  • causes pain. 

Histamine

  • acts on H1 receptors to cause
    • dilation of arterioles 
    • increased vascular permeability of venules


5. Which one of the listed statements is the best histologic definition of an abscess?

    a. A circumscribed collection of neutrophils with necrotic cellular debris

    b. A localized defect that results from the sloughing of necrotic inflammatory tissue from the surface         of an organ

    c. A localized proliferation of fibroblasts and small blood vessels

    d. An aggregate of two or more activated macrophages

    e. The excessive secretion of mucus from a mucosal surface


ANS: A

Note:

B. Ulcer

C. Granulation tissue

D. Granuloma

E. Catarrhal (phlegmonous or coryzal) inflammation, such as seen with a runny nose


Other morphologic patterns of inflammation include:

Fibrinous inflammation: 

  • Associated with the deposition of fibrin in body cavities, which subsequently stimulates coagulation. 
  • Histologically, fibrin is seen as amorphic eosinophilic material. 
  • Fibrinous inflammation within the pericardial cavity (fibrinous pericarditis) produces a characteristic “bread-and-butter” appearance grossly. 

Serous inflammation:

  • Produces a thin fluid, such as is present in skin blisters or body cavities. 
  • VS fibrinous inflammation: Not enough fibrinogen present in serous inflammation to form fibrin.

Pseudomembranous inflammation: 

  • formation of necrotic membranes on mucosal surfaces. 
  • Clostridium difficile
    •  “mushroom-shaped” pseudomembrane in the colon of people taking broad-spectrum antibiotics, and C. diphtheriae, which produces a pseudomembrane in the larynx.


6. A 25-year-old female presents with a history of losing four pregnancies in the past 5 years. She also has a history of recurrent pains in her legs secondary to recurrent thrombosis. Her symptoms are most likely due to a deficiency of

    a. PA inhibitors

    b. Protein C

    c. Plasmin

    d. Thrombin

    e. C′1 inactivator


ANS: B

Note:

Differential diagnosis of recurrent spontaneous abortions in women: 

  • Deficiencies of protein C and protein S
  • Presence of the lupus anticoagulant, which is part of the anti-phospholipid syndrome.


Two important control points of the coagulation cascade are the fibrinolytic system and certain plasma protease inhibitors. 

Fibrinolytic system: Plasmin

  • converted from plasminogen by either factor XII or a plasminogen activator (PA). 
    • Examples of PAs: tissue plasminogen activator (tPA), urokinase plasminogen activator, and streptokinase. 
  • Plasmin splits fibrin and also degrades both fibrinogen and coagulation factors VIII and V. 

Plasma protease inhibitors: Antithrombin III and protein C.
  • Antithrombin III: inhibits thrombin, XIIa, XIa, Xa, and IXa, 
  • Protein C inhibits Va and VIIIa. 
  • Deficiencies of antithrombin III, protein C, or protein S → Hypercoagulability of the blood → thrombosis
    • Hypercoagulability may be a primary (genetic) or secondary abnormality.
      • Primary hypercoagulable (genetic)
        • deficiencies of antithrombin III, protein C, or protein S. 
        • Associated with recurrent thromboembolism in early adult life and recurrent spontaneous abortions in women. 
      • Secondary hypercoagulable 
        • severe trauma, burns, disseminated cancer, and pregnancy.
        • Lower risk factors: age, smoking, and obesity. 
      • Some patients with high titers of autoantibodies against anionic phospholipids such as cardiolipin (the antibody being called a lupus anticoagulant) have a high frequency of arterial and venous thrombosis. 

7. Evaluation of a pedigree for a certain abnormality reveals the following information: there are skipped generations with male-to-male transmission; females are affected at the same rate as are males; and the disease is produced in the homozygous state, while heterozygous individuals are carriers.

What is the inheritance pattern for this disorder?

    a. Autosomal dominant

    b. Autosomal recessive

    c. X-linked dominant

    d. X-linked recessive

    e. Mitochondrial


ANS: B

Note:

A. symptoms manifested in the heterozygous state, males and females affected equally, and vertical transmission.

B. homozygous state, horizontal transmission, males and females affected equally, and common occurrence of complete penetrance and consanguineous relations. 

C. no male-to-male transmission, no skipped generations, females are affected twice as often as males.

D. no male-to-male transmission, affected male does not transmit the disease to his sons, but all daughters are carriers, sons of carrier females have a one in two chance of the disorder, but all daughters are asymptomatic, and the trait occurs in maternal uncles and in male cousins descended from the mother’s sisters (oblique transmission).


8. A 23-year-old female presents with progressive bilateral loss of central vision. You obtain a detailed family history from this patient and produce the associated pedigree (dark circles or squares indicate affected individuals).

Which of the following transmission patterns is most consistent with this patient’s family history?

【Pathology question】General Pathology Q8

    a. Autosomal recessive

    b. Autosomal dominant

    c. X-linked recessive

    d. X-linked dominant

    e. Mitochondrial


ANS: E

Note: This maternal origin means that mothers transmit all of the mtDNA to both male and female offspring, but only the daughters transmit it further. No transmission occurs through males.

Leber hereditary optic neuropathy (LHON), which is characterized by progressive bilateral loss of central vision and usually occurs between 15 and 35 years of age. 

Other examples of mitochondrial inheritance include mitochondrial myopathies, which are characterized by the presence in muscle of mitochondria having abnormal sizes and shapes. These abnormal mitochondria may result in the histologic appearance of the muscle as ragged red fibers. Electron microscopy reveals the presence within large mitochondria of rectangular crystals that have a “parking lot” appearance.


9. A 10-month-old baby is being evaluated for visual problems and motor incoordination. Examination of the child’s fundus reveals a bright “cherry red spot” at the macula. Talking to the family of this visually impaired 10-month-old infant, you find that they are Jewish and their family is from the eastern portion of Europe (Ashkenazi Jews). Based on this specific family history, which one of the following enzymes is most likely to be deficient in this infant?

    a. Aryl sulfatase

    b. β-glucocerebrosidase

    c. Hexosaminidase A

    d. Hexosaminidase B

    e. Sphingomyelinase


ANS: C

Note:

A. Metachromatic leukodystrophy (MLD)

B. Type I Gaucher’s disease

D. Sandhoff’s disease

E. Niemann-Pick disease


Lysosomal storage diseases:

  • abnormal accumulation of sphingolipids (SLs)
  • eg: Ganglion cells within the retina, particularly at the periphery of the macula, may become swollen with excess sphingolipids. 
    • Affected area of the retina appears pale when viewed through an ophthalmoscope. 
    • Normal color of the macula appears more red than normal.----- Cherry red spot or a cherry red macula. 
      • Substances that may produce this cherry red spot include 
        • Sphingomyelin, which is increased in individuals with Niemann- Pick disease
        • Gangliosides, which may be increased in individuals with Tay-Sachs disease, Sandhoff’s disease, or GM1 gangliosidosis.
  • eg. Tay-Sachs disease and type I Gaucher disease (autosomal recessive genes)
    • Ashkenazi Jews mostly of the Jewish faith, from Eastern Europe. 
      • marry other members of the faith. 
    • Tay-Sachs disease: deficiency of hexosaminidase A (deficiency of the α subunit)
      • Accumulation of GM2 ganglioside occurs within many tissues, including the heart, liver, spleen, and brain. 
      • Electron microscopy reveals cytoplasmic whorled lamellar bodies within lysosomes. 
      • Most severe form is the infantile type. 
        • Patients develop mental retardation, seizures, motor incoordination, and blindness (amaurosis), and usually die by the age of 3 years.
    • Sandhoff’s disease: deficiency of hexosaminidase A and B (deficiency of the β subunit)
        • Hexosaminidase A is composed of an α subunit and a β subunit. 
        • Hexosaminidase B is composed of two β subunits.
    • Type I Gaucher’s disease 
      • deficiency of β-glucocerebrosidase. 
      • Patients may have increased serum levels of acid phosphatase (an enzyme that is typically found in the prostate), erythrocytes, and platelets. 
      • Patients with Gaucher’s disease have accumulation of excess glucocerebrosides within phagocytic cells, not ganglion cells. 

10. A 4-year-old male with mental retardation, self-mutilation, and hyperuricemia is likely to have a deficiency of an enzyme involved in the

    a. Conversion of homogentisic acid to methylacetoacetate

    b. Degradation of galactocerebroside

    c. Breakdown of branched-chain amino acids

    d. Recycling of guanine and hypoxanthine

    e. Synthesis of UMP and CTP


ANS: D

Note:

A. Deficiency of homogentisic oxidase, which is involved in the conversion of homogentisic acid to methylacetoacetate, is associated with alkaptonuria. 

B. Krabbe’s disease

C. Maple syrup urine disease

Purine synthesis involves adding carbons and nitrogens to ribose 5-phosphate (R5P), which is a product of the hexose monophosphate (HMP) shunt. R5P is then converted to ribose phosphate pyrophosphate (RPPP), which is subsequently converted to 5′-phosphoribosylamine, the latter step being the committed step in purine nucleotide biosynthesis. Through a series of steps RPPP is converted to inosine 5′-monophosphate (IMP). Several of these biochemical steps involve transferring methyl groups from folate. This is important because folate analogues, such as methotrexate, inhibit DNA synthesis, especially in rapidly growing tumor cells, by inhibiting purine synthesis. Finally IMP is converted into either AMP or GMP. 

These last biochemical steps are also connected to biochemical reactions that involve 
  • Adenosine deaminase: deficient in autosomal recessive form of SCID
  • Hypoxanthine-guanine phosphoribosyl transferase (HGPRT):
    • an enzyme of the purine salvage pathway for recycling guanine and 
    • deficient in X-linked recessive disorder Lesch-Nyhan syndrome. 
      • excess uric acid production, which may produce symptoms of gout, mental retardation, spasticity, self-mutilation, and aggressive behavior.


11. Adult individuals with the karyotype 47, XXY would most likely have which one of the following sets of laboratory serum values?

                                Follicle-Stimulating         Luteinizing
          Testosterone     Hormone (FSH)         Hormone (LH)     Estradiol

    a.     Increased            Increased                 Increased             Increased

    b.     Increased            Decreased               Decreased            Decreased

    c.     Decreased           Increased                Increased              Decreased

    d.     Decreased           Increased                 Increased            Increased

    e.     Decreased           Decreased                 Decreased          Decreased


ANS: D

Note:

Klinefelter’s syndrome

  • male hypogonadism or testicular dysgenesis
  • 47, XXY. The extra X is from the mother in most cases, 
  • associated with increased maternal age. 
  • The hypogonadism causes decreased testosterone levels, which leads to eunuchoidism, lack of secondary male characteristics, and a female distribution of hair. 
    • Tall: due to delayed fusion of the epiphysis from a lack of testosterone. 
    • High voice and gynecomastia
    • Increased incidence of breast cancer 
    • Blood levels of plasma follicle-stimulating hormone (FSH), luteinizing hormone (LH), and estradiol are all increased. 
    • Patients have small, firm, atrophic testes, histologic sections of which reveal atrophy, Leydig cell hyperplasia, sclerosis of the tubules, and lack of sperm production. 
    • Slight decrease in IQ, but they are not severely mentally retarded.


12. A young boy is being evaluated for developmental delay, mild autism, and mental retardation. Physical examination reveals the boy to have large, everted ears and a long face with a large mandible. He is also found to have macroorchidism (large testes), and extensive workup reveals multiple tandem repeats of the nucleotide sequence CGG in his DNA. Which one of the following is the correct diagnosis for this patient?

    a. Fragile X syndrome

    b. Huntington’s chorea

    c. Myotonic dystrophy

    d. Spinal-bulbar muscular atrophy

    e. Ataxia-telangiectasia


ANS: A

Note:

Fragile X syndrome, Huntington’s disease, myotonic dystrophy, and spinal and bulbar muscular atrophy are characterized by long repeating sequences of three nucleotides. 

Fragile X syndrome:

  • more common in males than females
  • most common causes of familial mental retardation. 
  • Additional clinical features:
    • Developmental delay
    • A long face with a large mandible
    • Large everted ears
    • Large testicles (macroorchidism). 
  • Multiple tandem repeats of the nucleotide sequence CGG on the X chromosome. 

Normally these repeats average up to 50 in number, but in patients with fragile X syndrome there are more than 230 repeats. This number of repeats is called a full mutation. Normal transmitting males (NTMs) and carrier females have between 50 and 230 CGG repeats. This number of repeats is called a pre-mutation. During oogenesis, but not spermatogenesis, premutations can be converted to mutations by amplification of the triplet repeats. This explains the much higher incidence of mental retardation in grandsons rather than brothers of normal transmitting males (Sherman’s paradox), as the premutation is amplified in females but not in males. Since the premutation is not amplified in males, no daughters of NTMs are affected. An additional finding associated with these repeat units is anticipation, which refers to the fact that the disease is worse in subsequent generations.


13. A biopsy of an enlarged salivary gland from an individual with Sjögren’s syndrome is most likely to histologically reveal an extensive infiltrate of

    a. Basophils

    b. Eosinophils

    c. Epithelioid cells

    d. Lymphocytes

    e. Neutrophils


ANS: D

Note:

Sjögren’s syndrome:

  • dryness of the mouth (xerostomia)  
  • dryness of eyes (keratoconjunctivitis sicca). 
  • rheumatoid arthritis (RA), SLE, or systemic sclerosis. 

Primary form: Increased frequency of HLA-DR3, while association with RA shows a positive correlation with HLA-DR4. Anti-SSB antibodies are fairly specific, anti-SSA less so, and both may occur in SLE; rheumatoid factor is often present. Glomerular lesions are very rare, but a mild tubulointerstitial nephritis is quite common and may result in renal tubular acidosis. In addition to the usual dense, lymphoplasmacytic infiltrate of salivary glands, the lymph nodes may show a “pseudolymphomatous” appearance. True B cell lymphomas have developed with increased frequency in Sjögren’s syndrome.


14. An 8-month-old male infant is admitted to the hospital because of a bacterial respiratory infection. The infant responds to appropriate antibiotic therapy, but is readmitted several weeks later because of severe otitis media. Over the next several months, the infant is admitted to the hospital multiple times for recurrent bacterial infections. Workup reveals extremely low serum antibody levels. The infant has no previous history of viral or fungal infections. The most likely diagnosis for this infant is

    a. Isolated IgA deficiency

    b. Chronic granulomatous disease

    c. DiGeorge’s syndrome

    d. Wiskott-Aldrich syndrome

    e. X-linked agammaglobulinemia of Bruton


ANS: E

Note:

A. Immunodeficiency. 

  • Isolated IgA deficiency-----Most common form
  • due to a block in the terminal differentiation of B lymphocytes. 
  • Most patients are asymptomatic, but some develop chronic sinopulmonary infections. 
  • Patients are prone to developing diarrhea (Giardia infection) and also have an increased incidence of autoimmune disease, such as Hashimoto’s thyroiditis. 

B. Chronic granulomatous disease

  • neutrophils and macrophages have deficient H2O2 pro-duction due to abnormalities involving the enzyme NADPH oxidase. 
  • have frequent infections that are caused by catalase-positive organisms, such as S. aureus, 
    • because the catalase produced by these organisms destroys the little hydrogen peroxide that is produced. 

C. DiGeorge’s syndrome

  • T cell–deficiency disorder that results from hypoplasia of the thymus due to abnormal development of the third and fourth pharyngeal pouches. 
  • The parathyroid glands are also abnormal, may develop hypocalcemia and tetany
  • Congenital heart defects are also present. 

D. Wiskott-Aldrich syndrome 

  • X-linked recessive disorder
  • thrombocytopenia, eczema, and immune deficiency
  • Immune abnormalities: 
    • Progressive loss of T cell function and decreased IgM. 
    • The other immunoglobulin levels are normal or increased. 
    • Decreased numbers of lymphocytes in the peripheral blood and paracortical (T cell) areas of lymph nodes. 
    • Both cellular and humoral immunity are affected, and, because patients fail to produce antibodies to polysaccharides, they are vulnerable to infections with encapsulated organisms.

E. X-linked agammaglobulinemia of Bruton

  • B cells are absent but numbers and function of T cells are normal. 
  • This abnormality results from defective maturation of B lymphocytes beyond the pre-B stage. 
  • This maturation defect leads to decreased or absent numbers of plasma cells, 
  • therefore immunoglobulin levels are markedly decreased. 
  • Male infants with Bruton’s disease 
    • begin having trouble with recurrent bacterial infections at about the age of 9 months, which is when maternal antibodies are no longer present in the affected infant. 
  • Therapy for Bruton’s disease: IV gamma globulin.


15. During a routine physical examination, a 45-year-old male is found to have microscopic hematuria. Further workup reveals a 4.5-cm mass in the upper pole of his right kidney. This mass is resected and reveals a tumor composed of a uniform population of cells with clear cytoplasm. Mitoses are not found. Further workup fails to reveal the presence of any metastatic disease. Based on all of these findings, which of the following best characterizes this tumor? (Note: assume that a low stage has a better prognosis than a high stage, and a low grade has a better prognosis than a high grade. Also assume that renal tumors composed of cells with clear cytoplasm that are larger than 2.0 cm in diameter are malignant.)

            Tumor
        Aggressiveness                 Grade                     Stage

    a.     Benign                 Not applicable         Not applicable

    b.     Malignant                     Low                         Low

    c.     Malignant                     Low                         High

    d.     Malignant                     High                         Low

    e.     Malignant                     High                         High


ANS: B

Note:

Grading is done histologically, based on the 

  • Histologic degree of differentiation of the tumor cells 
    • Indicators of the aggressiveness of the malignant neoplasm
    • Cancers are generally classified as grades I (Low) through IV (High). 
      • Grades I and II: less aggressive and have a better prognosis, 
      • Grades III and IV: more aggressive and have a worse prognosis. 
  • On the number of mitoses that are present


Staging is done clinically, based on the 

    • Size of the primary lesion, 
    • Presence of lymph node metastases
    • Presence of bloodborne metastases. 
  • These characteristics are determined by clinical means: TNM classification
    • T: Tumor size,
    • N: Presence of lymph node metastases
    • M: Presence of non–lymph node metastases. 


16. A 35-year-old male living in a southern region of Africa presents with increasing abdominal pain and jaundice. He has worked as a farmer for many years, and sometimes his grain has become moldy. Physical examination reveals a large mass involving the right side of his liver, and a biopsy specimen from this mass confirms the diagnosis of liver cancer (hepatocellular carcinoma). The pathogenesis of this tumor involves which of the following substances?

    a. Aflatoxin B1

    b. Direct-acting alkylating agents

    c. Vinyl chloride

    d. Azo dyes

    e. β-naphthylamine


ANS: A

Note:

Many chemicals are associated with an increased incidence of malignancy. These substances are called chemical carcinogens. 

Although there are direct-acting chemical carcinogens, such as the direct-acting alkylating agents that are used in chemotherapy, most organic carcinogens first require conversion to a more reactive compound. 

Polycyclic aromatic hydrocarbons, aromatic amines, and azo dyes must be metabolized by cytochrome P450–dependent mixedfunction oxidases to active metabolites. 

Vinyl chloride is metabolized to an epoxide and is associated with angiosarcoma of the liver, not hepatocellular carcinoma. 

Azo dyes, such as butter yellow and scarlet red, are metabolized to active compounds that have induced hepatocellular cancer in rats, but no human cases have been reported. 

β-naphthylamine is an exception to the general rule involving cytochrome P450, as the hydrolysis of the nontoxic conjugate occurs in the urinary bladder by the urinary enzyme glucuronidase. 

In the past there has been an increase in bladder cancer in workers in the aniline dye and rubber industries who have been exposed to these compounds. 

Aflatoxin B1, a natural product of the fungus Aspergillus flavus, is metabolized to an epoxide. The fungus can grow on improperly stored peanuts and grains and is associated with the high incidence of hepatocellular carcinoma in some areas of Africa and the Far East. Hepatitis B virus is also highly associated with liver cancer in these regions.


17. A 59-year-old male is found to have a 3.5-cm mass in the right upper lobe of his lung. A biopsy of this mass is diagnosed as a moderately differentiated squamous cell carcinoma. Workup reveals that no bone metastases are present, but laboratory examination reveals that the man’s serum calcium levels are 11.5 mg/dL. This patient’s paraneoplastic syndrome is most likely the result of ectopic production of

    a. Parathyroid hormone

    b. Parathyroid hormone–related peptide

    c. Calcitonin

    d. Calcitonin-related peptide

    e. Erythropoietin


ANS: B

Note:

Symptoms not caused by either local or metastatic effects of tumors are called paraneoplastic syndromes. Bronchogenic carcinomas are associated with the development of many different types of paraneoplastic syndromes. These syndromes are usually associated with the secretion of certain substances by the tumor cells. 

  • Ectopic secretion of ACTH: Cushing’s syndrome, 
  • Ectopic secretion of antidiuretic hormone (syndrome of inappropriate ADH secretion): hyponatremia. 
    • Hypocalcemia may result from the production of calcitonin

Hypercalcemia may result from the production of parathyroid hormone–related peptide (PTHrP), which is a normal substance produced locally by many different types of tissue.

PTHrP is distinct from parathyroid hormone (PTH). Therefore, patients with this type of paraneoplastic syndrome have increased calcium levels and decreased PTH levels. As a result of decreased PTH production, all of the parathyroid glands in these patients are atrophic. Other tumors associated with the production of PTHrP include clear cell carcinomas of the kidney, endometrial adenocarcinomas, and transitional carcinomas of the urinary bladder.

Lung cancers are also associated with multiple, migratory venous thromboses. This migratory thrombophlebitis is called Trousseau’s sign and is more classically associated with carcinoma of the pancreas. Hypertrophic osteoarthropathy is a syndrome consisting of periosteal new bone formation with or without digital clubbing and joint effusion. It is most commonly found in association with lung carcinoma, but it also occurs with other types of pulmonary disease. Erythrocytosis is associated with increased erythropoietin levels and some tumors, particularly renal cell carcinomas, hepatocellular carcinomas, and cerebellar hemangioblastomas. It is not particularly associated with bronchogenic carcinomas.


18. A 22-year-old female presents with the sudden onset of a high fever, a diffuse erythematous skin rash, and shock. She started menstruating at age 13 and for several years has used tampons. Which one of the following is the most likely diagnosis for this individual’s illness?

    a. Erysipelas caused by Streptococcus pyogenes

    b. Fifth disease caused by human parvovirus B19

    c. Scarlet fever caused by S. pyogenes

    d. Secondary syphilis caused by Treponema pallidum

    e. Toxic shock syndrome caused by Staphylococcus aureus


ANS: E

Note:

Toxic shock syndrome caused by infection with certain types of S. aureus that secrete the toxin toxic shock syndrome toxin 1 (TSST-1). This toxin is a type of superantigen that binds both class II MHC and TCR outside of the normal antigen-binding groove. As such, this toxin can react with up to 10% of peripheral T cells, which leads to massive T cell activation and shock. Clinically, toxic shock syndrome is most often seen in women who use certain tampons that have been colonized with Staphylococcus.


19. Several days after exploring a cave in eastern Kentucky, a 39-year-old female develops shortness of breath and a low-grade fever. Chest x-rays reveal several irregular areas in both upper lung fields along with enlarged hilar and mediastinal lymph nodes. A biopsy of one of these lymph nodes reveals granulomatous inflammation. Multiple small yeasts surrounded by clear zones are seen within macrophages. Which one of the following organisms is most likely responsible for this individual’s disease?

    a. Aspergillus species

    b. Blastomyces dermatitidis

    c. C. albicans

    d. Histoplasma capsulatum

    e. Mucor


ANS: D

Note:

Histoplasmosis is caused by the reproductive cells (spores) of the fungus Histoplasma capsulatum. They float into the air when dirt or other material is disturbed. The fungus thrives in damp soil that's rich in organic material, especially the droppings from birds and bats. Although H. capsulatum commonly produces asymptomatic primary disease, it can result in granulomatous inflammation, especially granulomatous lung disease. Multiple small yeasts surrounded by clear zones may be found within the cytoplasm of macrophages.

A. Produce several clinical disease states, including allergic aspergillosis, systemic aspergillosis, and aspergilloma. Typically Aspergillus species are seen in tissue as acute angle–branching septate hyphae; however, they may form fruiting bodies in cavities, such as within cystic cavities of the lungs. There they may form a large mass called a fungus ball or aspergilloma.

B. Blastomycosis is a chronic granulomatosis disease caused by a dimorphic fungus, B. dermatitidis. In tissues this fungus is seen as a thick-walled yeast having broad-based budding. Without the budding, Blastomyces may be mistaken for Cryptococcus. The infection, also known as Gilchrist’s disease, is seen in individuals living in the Ohio and Mississippi Valley areas and is usually confined to the lungs.

C. Candida species, which frequently cause human infections, grow as yeasts, elongated chains of yeast without hyphae (pseudohyphae), or septate hyphae. 

E. Mucocutaneous candidal infections can produce white plaques called thrush. Mucormycosis (zygomycosis) is a disease caused by “bread mold fungi” such as Rhizopus, Mucor, and Absidia species. These infections typically occur in neutropenic patients or diabetics. One form of the disease, typically found in diabetics, is called rhinocerebral mucormycosis and is characterized by facial pain, headache, changing mental status, and a blood-tinged nasal discharge. Tissue sections reveal characteristic broad, nonseptate, right angle–branching hyphae.


20. The use of broad-spectrum antibiotics can produce a bleeding diathesis characterized by hematomas, hematuria, melena, and bleeding from the gums by decreasing the normal gut flora and inducing a deficiency of

    a. Vitamin A

    b. Vitamin B1

    c. Vitamin B6

    d. Vitamin C

    e. Vitamin K


ANS: E

Note: Up to 50% of the vitamin K needed by the body is provided by the normal bacteria of the GI tract. Vitamin K is required for the posttranslational conversion of glutamyl residues in some proteins into γ-carboxylates. It participates in the hepatic carboxylation of four procoagulants (factors II, VII, IX, and X) and plasma proteins C and S. For these four proclotting factors, this γ-carboxylation provides the calcium-binding sites necessary for the calcium-dependent interaction with a phospholipid surface. A deficiency of vitamin K, which can result from fat malabsorption, broad-spectrum antibiotics, or diffuse liver disease, produces a bleeding diathesis characterized by hematomas, ecchymoses, hematuria, melena, and bleeding from the gums. Laboratory tests reveal an increased bleeding with prolonged PT and PTT and normal bleeding time.


21. Hypoxia decreases cellular levels of ATP and inhibits the normal function of the plasma membrane ouabain-sensitive Na-K-ATPase pump.

Which one of the listed changes will result from decreased function of this membrane ion pump?

               

【Pathology question】General Pathology Q21

ANS: C

Note: Having low oxygen levels in your blood is called hypoxemia. 

Damage to cells may result in reversible or irreversible injury. Common mediators of cell injury include chemicals, toxins, free radicals, and decreased oxygen delivery by the blood. Decreased blood flow (ischemia) decreases ATP production by aerobic cellular processes because of the deficiency of oxygen. This results in decreased oxidative phosphorylation by mitochondria, which decreases the functioning of the ATP-dependent sodium pump of the plasma membrane.

This decreases the efflux of sodium ions outside the cell and decreases the influx of potassium out of the cell, which increases the sodium ions inside the cell and increases the potassium ions outside the cell

The resultant net gain of intracellular ions causes isosmotic water accumulation and hydropic swelling (cloudy swelling) of the cell and the organelles of the cell. 

Decreased aerobic respiration by mitochondria also increases anaerobic glycolysis, which decreases intracellular pH by increasing lactic acid production (lactic acidosis). The decreased pH causes chromatin clumping and may activate lysosomal enzymes. Additionally, ribosomes can dissociate from the endoplasmic reticulum (RER), which decreases protein production by cell.

All of these changes that result from hypoxia are characteristic of reversible cellular injury, as they are reversible if blood flow and oxygen supply are restored.


22. A 54-year-old male develops a thrombus in his left anterior descending coronary artery. The area of myocardium supplied by this vessel is irreversibly injured. The thrombus is destroyed by the infusion of streptokinase, which is a plasminogen activator, and the injured area is reperfused. The patient, however, develops an arrhythmia and dies. An electron microscopic (EM) picture taken of the irreversibly injured myocardium reveals the presence of large, dark, irregular amorphic densities within mitochondria, which are referred to as

    a. Apoptotic bodies

    b. Flocculent densities

    c. Myelin figures

    d. Psammoma bodies

    e. Russell bodies


ANS: B

Note:

With prolonged ischemia, certain cellular events occur that are not reversible, even with restoration of oxygen supply. These cellular changes are referred to as irreversible cellular injury. This type of injury is characterized by severe damage to mitochondria (vacuole formation), extensive damage to plasma membranes and nuclei, and rupture of lysosomes. Severe damage to mitochondria is characterized by the influx of calcium ions into the mitochondria and the subsequent formation of large, flocculent densities within the mitochondria.

These flocculent densities are characteristically seen in irreversibly injured myocardial cells that undergo reperfusion soon after injury. Less severe changes in mitochondria, such as mitochondrial swelling, are seen with reversible injury. 

A. Cytochrome c released from damaged mitochondria can induce apoptosis, a process through which irreversibly injured cells can shrink and increase the eosinophilia of their cytoplasm. These shrunken apoptotic cells (apoptotic bodies) may be engulfed by adjacent cells or macrophages. 

C. Myelin figures are derived from plasma membranes and organelle membranes and can be seen with either reversible or irreversible injury. 

D. Psammoma bodies are small, laminated calcifications

E. Russell bodies are round, eosinophilic aggregates of immunoglobulin.


23. Which one of the following microscopic associations concerning hepatocytes is correct?

a. Clear cytoplasmic material that is oil red O–negative but PAS-positive is most likely to be cholesterol

b. Clear cytoplasmic material that is oil red O–positive but PAS-negative is most likely to be lipofuscin

c. Clear nuclear material that is Prussian blue–positive is most likely to be hemosiderin

d. Yellow-brown granular cytoplasmic material that is Prussian blue–negative is most likely to be bile

e. Yellow-brown granular cytoplasmic material that is Prussian blue–positive is most likely to be melanin


ANS: D

Note: 

The differential of brown (or yellow-brown) granules in hepatocytes seen with routine hematoxylin and eosin (H&E) stain includes hemosiderin, bile, and lipofuscin

The special histologic stain for hemosiderin, which contains iron, is Prussian blue. Causes of excess iron deposition in the liver include hemosiderosis, which can result from excessive blood transfusions, and familial hemochromatosis, which results from excessive iron absorption from the gut. 

Excess bile in the liver can be seen with jaundice.

Lipofuscin deposition is seen with aging, cachexia, and severe malnutrition. Lipofuscin is a “wear-and-tear” pigment composed of lipids and phospholipids from lipid peroxidation by free radicals of lipids of subcellular membranes. It does not stain with Prussian blue stain, trichrome stain, oil red O stain, or periodic acid–Schiff (PAS) stain, but is seen best with electron microscopy. 

The differential of clear spaces in cytoplasm of cells as seen with light microscopy includes glycogen, lipid, and water. 

Glycogen is a PAS-positive material that is diastasesensitive,

while lipid is oil red O–positive. 

Accumulation of water is called hydropic change. There is no special stain for water.


24. A 48-year-old male who has a long history of excessive drinking presents with signs of alcoholic hepatitis. Microscopic examination of a biopsy of this patient’s liver reveals irregular eosinophilic hyaline inclusions within the cytoplasm of the hepatocytes. These eosinophilic inclusions are composed of

    a. Immunoglobulin

    b. Excess plasma proteins

    c. Prekeratin intermediate filaments

    d. Basement membrane material

    e. Lipofuscin


ANS: C

Note:

Hyalin is a nonspecific term that is used to describe any material, inside or outside the cell, that stains a red homogenous color with the routine hematoxylin and eosin (H&E) stain. There are many different substances that have the appearance of hyalin. 

Alcoholic hyaline inclusions (Mallory bodies) are irregular eosinophilic hyaline inclusions that are found within the cytoplasm of hepatocytes. Mallory bodies are composed of prekeratin intermediate filaments. They are a nonspecific finding and can be found in patients with several diseases other than alcoholic hepatitis, such as Wilson’s disease, and in patients who have undergone bypass operations for morbid obesity.

A. Immunoglobulins may form intracytoplasmic or extracellular oval hyaline bodies called Russell bodies. 

B. Excess plasma proteins may form hyaline droplets in proximal renal tubular epithelial cells or hyaline membranes in the alveoli of the lungs (hyaline membrane disease). 

D. The hyalin found in the walls of arterioles of kidneys in patients with benign nephrosclerosis is composed of basement membranes and precipitated plasma proteins.

E. Lipofuscin is an intracytoplasmic aging pigment that has a yellow-brown, finely granular appearance with H&E stains. Its appearance does not resemble that of hyaline material.


25. A 38-year-old female presents with intermittent pelvic pain. Physical examination reveals a 3-cm mass in the area of her right ovary. Histologic sections from this ovarian mass reveal a papillary tumor with multiple, scattered small, round, laminated calcifications. These structures are most likely the result of

    a. Apoptosis

    b. Dystrophic calcification

    c. Enzymatic necrosis

    d. Hyperparathyroidism

    e. Metastatic calcification


ANS: B

Note:

Calcification within tissue can be classified as being dystrophic or metastatic.

  • Dystrophic calcification is characterized by calcification in abnormal (dystrophic) tissue
    • eg. calcification within severe atherosclerosis
    • calcification of damaged or abnormal heart valves
    • calcification within tumors
      • Small (microscopic) laminated calcifications within tumors are called psammoma bodies and are due to single-cell necrosis. Psammoma bodies are characteristically found in papillary tumors, such as papillary carcinomas of the thyroid and papillary tumors of the ovary (especially papillary serous cystadenocarcinomas), but they can also be found in meningiomas or mesotheliomas.  

Metastatic calcification is characterized by calcification in normal tissue. 

Dystrophic calcification within tumors of the central nervous system (CNS), which can be seen with x-rays, is useful in the differential diagnosis of these CNS tumors.Dystrophic calcification within tumors of the central nervous system (CNS), which can be seen with x-rays, is useful in the differential diagnosis of these CNS tumors.

For example, calcification of a tumor of the cortex in an adult is suggestive of an oligodendroglioma, while calcification of a hypothalamus tumor is suggestive of a craniopharyngioma. 

Additional periventricular calcification in children is most commonly caused by infection with cytomegalovirus (CMV) or toxoplasmosis. With dystrophic calcification the serum calcium levels are normal, while with metastatic calcification the serum calcium levels are elevated (hypercalcemia). 

Causes of hypercalcemia include certain paraneoplastic syndromes, such as secretion of parathyroid hormone–related peptide, hyperparathyroidism, iatrogenic causes (drugs), immobilization, multiple myeloma, increased milk consumption (milk-alkali syndrome), and sarcoidosis.


26. After binding to Fas ligand (CD95L), Fas (CD95) self-associates and activates Fas-associated death domain protein (FADD), which in turn induces apoptosis by stimulating

a. bcl-2

b. Caspase 8

c. Cytochrome a3

d. Cytochrome p450

e. Elastase 6


ANS: B

Note:

Apoptosis is a distinctive pattern of cell death that is described as a “programmed suicide” process of cells through which stimulation of endogenous endonucleases causes cleavage of nuclear chromatin. Apoptosis as originally defined is a purely morphologic process that differs from necrosis in several respects.

Apoptosis involves single cells, not large groups of cells, and with apoptosis the cells shrink and there is increased eosinophilia of cytoplasm. The shrunken apoptotic cells form apoptotic bodies, which may be engulfed by adjacent cells or macrophages. With apoptosis there is no inflammatory response, the cell membranes do not rupture, and there is no release of macromolecules. 

Importantly, apoptosis is an active process in which activation of endonucleases causes peripheral condensation of chromatin (the most characteristic feature) and formation of multiples of DNA base pair fragments (called a DNA “ladder”).

There are several mechanisms involved in apoptosis of cells. 👇

Several of these mechanisms involve substances found in either the bcl-2 oncogene family or the tumor necrosis family; the latter includes Fas and tumor necrosis factor (TNF). 

One mechanism of apoptosis involves cytochrome c being released into the cytoplasm from mitochondria via bax channels, which are upregulated by p53. Cytochrome c then binds to and activates apoptosis activating factor 1 (Apaf-1), which then stimulates a caspase cascade. The product of bcl-2 is normally located on the outer mitochondrial membrane, endoplasmic reticulum, and nuclear envelope. This product inhibits apoptosis by blocking bax channels and by binding to and sequestering Apaf-1. 

In a different mechanism, Fas ligand (FasL) is produced by immune cells and binds to Fas (CD95), which activates Fas-associated death domain protein (FADD), which in turn activates caspase 8. Cytotoxic T lymphocytes stimulate apoptosis by expressing FasL or secreting substances like perforin (which forms pores) or granzyme B. 

In one final example, TNF binds to TNF receptor 1 (TNFR-1) and the complex activates TNF-receptor-adaptor protein with a death domain (TRADD), which activates FADD.


Apoptosis is the type of cell death seen with embryonic development, death of immune cells, hormone-induced atrophy, and some bacterial toxins or viral infections. 

Examples of apoptosis of immune cells include the involution of the thymus with aging and the destruction of proliferating B cells in germinal centers of lymph nodes. 

Examples of apoptosis resulting from hormone-induced atrophy exclude the death of endometrial cells during menses, ovarian follicular atresia after menopause, and regression of the lactating breast after weaning. 

An example of a viral infection causing apoptosis is the formation of Councilman bodies in the livers of patients with viral hepatitis. It is important to note that abnormalities of genes involved in apoptosis may contribute to the formation of some malignancies; for example, activation of bcl-2 is associated with nodular B cell non-Hodgkin’s lymphomas.


27. A 49-year-old man develops an acute myocardial infarction because of the sudden occlusion of the left anterior descending coronary artery. The areas of myocardial necrosis within the ventricle can best be described as

    a. Coagulative necrosis

    b. Liquefactive necrosis

    c. Fat necrosis

    d. Caseous necrosis

    e. Fibrinoid necrosis


ANS: A

Note: The cause of cell injury and death may sometimes be inferred from the type of necrosis present. 

Coagulative necrosis, characterized by loss of the cell nucleus, acidophilic change of the cytoplasm, and preservation of the outline of the cell, is seen in sudden, severe ischemia of many organs. It is not present, however, in acute ischemic necrosis of the brain. Myocardial infarction resulting from the sudden occlusion of the coronary artery is a classic example of coagulative necrosis. 

B. In contrast, with liquefactive necrosis the dead cells are completely dissolved by hydrolytic enzymes. This type of necrosis can be seen in ischemic necrosis of the brain, but classically it is associated with acute bacterial infections. 

C. Fat necrosis, seen with acute pancreatic necrosis, is fat cell death caused by lipases.

D. Caseous necrosis is a combination of coagulative and liquefactive necrosis, but the necrotic cells are not totally dissolved and remain as amorphic, coarsely granular, eosinophilic debris. This type of necrosis grossly has the appearance of clumped cheese. It is classically seen in tuberculous infections. 

E. Fibrinoid necrosis is an abnormality seen sometimes in injured blood vessels where plasma proteins abnormally accumulate within the vessel walls. 

Gangrenous necrosis of extremities is also a combination of coagulative and liquefactive necrosis. In dry gangrene the coagulative pattern is predominate, while in wet gangrene the liquefactive pattern is predominate.


28. The degradation of intracellular organelles through the process in which autosomes combine with primary lysosomes to form autophagolysosomes is called

    a. Autophagy

    b. Heterophagy

    c. Heteroplasmy

    d. Homophagy

    e. Endocytosis


ANS: A

Note:

The primary function of lysosomes is to degrade macromolecules derived from either intracellular organelles (autophagy) or extracellular products (heterophagy). Primary lysosomes are cytoplasmic vacuoles that contain numerous acid hydrolases produced by the Golgi. These vacuoles combine either with vacuoles containing cellular components (autosomes) or with clathrin-coated endocytic vesicles that contain extracellular material (phagosomes). This fusion forms the secondary lysosome (multivesicular body, or phagolysosome) in which the macromolecules are degraded. If the extracellular material is a ligand coupled to a receptor, then the clathrin-coated endocytic vesicle is called a compartment of uncoupling of receptor and ligand (CURL), and the receptors are then recycled to the plasma membrane. The products of the normal lysosomal function are usually reutilized by the cell, but if the material is not digestible (e.g., lipofuscin or hemosiderin), the end result may be production of residual bodies, which may be removed from the cell by exocytosis.

A. The body's way of cleaning out damaged cells, in order to regenerate newer, healthier cells

B. Digestion in a fused vacuole and lysosome of material taken into a cell by phagocytosis or pinocytosis.

C. The presence of more than one mtDNA type in an individual


29. Histologic sections of an enlarged tonsil from a 9-year-old female reveal an increased number of reactive follicles containing germinal centers with proliferating B lymphocytes. Which one of the listed terms best describes this pathologic process?

    a. B lymphocyte hypertrophy

    b. Follicular dysplasia

    c. Follicular hyperplasia

    d. Germinal center atrophy

    e. Germinal center metaplasia


ANS: C

Note: There are many adaptive mechanisms of cells to persistent stimuli. 

Hypertrophy is an increase in the size of cells. 

  • eg. enlarged skeletal muscle in response to repeated exercise or anabolic steroid use and enlarged cardiac muscle in response to volume overload or hypertension. 

Hyperplasia is an increase in the number of cells. Hyperplasia may be the result of a physiologic response or a pathologic process. 

  • Examples of physiologic hyperplasia include the increased size of the female breast or uterus in response to hormones. 
  • Pathologic hyperplasia may be compensatory to some abnormal process, or it may be a purely abnormal process.
    • Examples of compensatory pathologic hyperplasia include the regenerating liver, increased numbers of erythrocytes in response to chronic hypoxia, and increased numbers of lymphocytes within lymph nodes in response to bacterial infections [follicular (nodular) hyperplasia].
    • Examples of purely pathologic hyperplasia include abnormal enlargement of the endometrium (endometrial hyperplasia) and the prostate (benign prostatic hyperplasia).

B. Dysplasia refers to disorganized growth and is characterized by the presence of atypical or dysplastic cells. Dysplasia can be seen in many organs, such as within the epidermis in response to sun damage (actinic keratosis), the respiratory tract, or the cervix (cervical dysplasia).

D. Atrophy is a decrease in the size and function of cells. Examples of atrophy include decreased size of limbs immobilized by a plaster cast or paralysis, or decreased size of organs affected by endocrine insufficiencies or decreased blood flow. 

E. Metaplasia is a term that describes the conversion of one histologic cell type to another. Examples of metaplasia include respiratory epithelium changing to stratified squamous epithelium (squamous metaplasia) in response to prolonged smoking, the normal glandular epithelium of the endocervix changing to stratified squamous epithelium (squamous metaplasia) in response to chronic inflammation, or the normal stratified squamous epithelium of the lower esophagus changing to gastric-type mucosa in response to chronic reflux. 


30. A patient presents with a large wound to his right forearm that is the result of a chain saw accident. You treat his wound appropriately and follow him in your surgery clinic at routine intervals. Initially his wound is filled with granulation tissue, which is composed of proliferating fibroblasts and proliferating new blood vessels (angiogenesis). A growth factor that is capable of inducing all the steps necessary for angiogenesis is

    a. Epidermal growth factor (EGF)

    b. Transforming growth factor α (TGF-α)

    c. Platelet-derived growth factor (PDGF)

    d. Basic fibroblast growth factor (FGF)

    e. Transforming growth factor β (TGF-β)


ANS: D

Note: Growth factors are chemicals that are associated with cell growth. For example, 

Fibroblast growth factor (FGF) can induce the growth and proliferation of fibroblasts. Additionally, one type of FGF, basic FGF, is capable of inducing all of the stages of angiogenesis (basement membrane and extracellular matrix degradation, endothelial migration, endothelial proliferation, and endothelial differentiation). 

The epidermal growth factor family includes epidermal growth factor (EGF) and transforming growth factor α (TGF-α). These substances can cause proliferation of many types of epithelial cells and fibroblasts. The EGF receptor is c-erb B1. 

Platelet-derived growth factor (PDGF), which is found in platelets, activated macrophages, endothelial cells, and smooth muscle cells, can cause migration and proliferation of fibroblasts, smooth muscle cells, and monocytes. 

TGF-β, produced by platelets, endothelial cells, T cells, and macrophages, is associated with fibrosis. In low concentrations it causes the synthesis and secretion of PDGF, but in high concentrations it inhibits growth due to inhibition of the expression of PDGF receptors.


31. The cardinal sign of inflammation called rubor is mainly the result of

    a. Decreased interstitial hydrostatic pressure

    b. Decreased vascular permeability of capillaries

    c. Increased vascular permeability of venules

    d. Vasoconstriction of muscular arteries

    e. Vasodilation of arterioles


ANS: E

Note: 

Inflammation can be defined as the reaction of vascularized living tissue to local injury. Celsus originally described four cardinal signs of inflammation: rubor (redness), tumor (swelling), calor (heat), and dolor (pain). Virchow later added a fifth sign, loss of function (functio laesa). Redness (rubor) and heat (calor) are primarily the result of increased blood flow secondary to vasodilation of arterioles. This vasodilation is mainly the result of prostaglandins (prostacyclin) and nitric oxide, but histamine and bradykinin also participate in this response. Swelling (tumor) results from fluid leaking into the interstitium, while pain (dolor) results from the secretion of bradykinin. Loss of function (functiolaesa) results from the swelling and pain.


32. During the early stages of the inflammatory response, histamineinduced increased vascular permeability is most likely to occur in

    a. Arteries

    b. Precapillary arterioles

    c. Capillaries

    d. Postcapillary venules

    e. Veins


ANS: D

Note: 

Acute inflammation causes fluid and cells to leak out of blood vessels into the interstitial tissue because of increased permeability of postcapillary venules. This increased vascular permeability results from either direct endothelial injury or contraction of endothelial cells. Substances that cause the latter include histamine (secreted from mast cells, basophils, and platelets), bradykinin, complement components (C3a and C5a), and leukotrienes (C4, D4, and E4). The result of this increased vascular permeability is that large amounts of fluid and cells from the blood can leak into the interstitial tissue. This inflammatory edema fluid, called an exudate, is characterized by a high protein content, numerous inflammatory cells (mainly neutrophils), abundant cellular debris, and a specific gravity greater than 1.020. In contrast to exudates, transudates result from either increased intravascular hydrostatic pressure or decreased osmotic pressure and are characterized by a low protein content, few cells, and a specific gravity less than 1.012.


33. Which one of the listed statements best describes the process called chemotaxis?

    a. Abnormal fusion of phagosomes to primary lysosomes

    b. Attachment of chemicals to extracellular material to increase phagocytosis

    c. Dilation of blood vessels by chemotherapeutic drugs

    d. Movement of cells toward a certain site or source

    e. Transmigration of cells from blood vessels into tissue


ANS: D

Note: Chemotaxis is the unidirectional migration of cells toward a certain site or source. 

The most significant chemotactic agents for neutrophils include bacterial products, complement components (particularly C5a), products of the lipoxygenase pathway (mainly leukotriene B4), and cytokines (particularly interleukin 8). 

These chemotactic factors bind to Gq receptors on the surface of neutrophils and activate phospholipase C, leading to the hydrolysis of phosphatidylinositol-4,5-biphosphate (PIP2) in the plasma membrane to inositol-1,4,5-triphosphate (IP3) and diacylglycerol (DAG). IP3 releases calcium from endoplasmic reticulum stores. DAG is the principal activator of protein kinase C, which phosphorylates many cytoplasmic proteins. These reactions result in increased calcium levels in the cytoplasm of neutrophils, which then stimulates the assembly of contractile elements in the cytoplasm of leukocytes (actin and myosin), causing movement.

These same chemotactic factors activate leukocytes, which results in increased production of arachidonic acid metabolites, activation of the respiratory (oxidative) burst, degranulation and secretion of lysosomal enzymes, and modulation of the leukocyte adhesion molecules.

In contrast, abnormal fusion of phagosomes to primary lysosomes is the principal defect in Chédiak-Higashi syndrome; attachment of chemicals to extracellular material to increase phagocytosis describes opsonins; and transmigration of cells from blood vessels into tissue refers to diapedesis.


34. A 3-year-old boy presents with recurrent bacterial and fungal infections primarily involving his skin and respiratory tract. Physical examination reveals the presence of oculocutaneous albinism. Examination of a peripheral blood smear reveals large granules within neutrophils, lymphocytes, and monocytes. The total neutrophil count is found to be decreased. Further workup reveals ineffective bactericidal capabilities of neutrophils due to defective fusion of phagosomes with lysosomes. What is the correct diagnosis?

    a. Ataxia-telangiectasia

    b. Chédiak-Higashi syndrome

    c. Chronic granulomatous disease

    d. Ehlers-Danlos syndrome

    e. Sturge-Weber syndrome


ANS: B

Note:

Chédiak-Higashi syndrome is an autosomal recessive disorder characterized by the abnormal fusion of phagosomes with lysosomes, which results in ineffective bactericidal capabilities of neutrophils and monocytes. These abnormal leukocytes develop giant intracytoplasmic lysosomes. Abnormal formation of melanosomes in these individuals results in oculocutaneous albinism. Most of these patients eventually develop an “accelerated phase” in which an aggressive lymphoproliferative disease, possibly the result of an Epstein-Barr viral infection, results in pancytopenia and death.

A. A chromosome instability syndrome that is characterized by increased sensitivity to x-rays (causing a markedly increased risk of lymphoid malignancies), recurrent infections, oculocutaneous telangiectasias (dilated blood vessels), and cerebellar ataxia. 

C. An X-linked recessive disorder characterized by recurrent bacterial infections due to deficient NADPH oxidase. 

D. Results from many different defects in formation of collagen and is generally characterized by fragile skin and hypermobile joints.

E. Characterized by capillary-venous malformation of leptomeninges and superficial cortex of one cerebral hemisphere with ipsilateral port-wine stains (nevus flammeus) in the trigeminal region of the face.


35. Which of the following laboratory findings is most suggestive of activation of the alternate complement system rather than the classic complement system?

【Pathology question】General Pathology Q35

ANS: B

Note: The complement system is a cascade of plasma proteins whose basic function is the direct lysis of cells, attraction of leukocytes to sites of inflammation (chemotaxis), and activation of leukocytes. The complement system can be activated by one of two basic pathways. 

The classic pathway is initiated by antigen-antibody (immune) complexes binding to C1. The antibodies that are involved in forming these complement-activating immune complexes are IgM and IgG (subtypes 1, 2, and 3). There are also some non-immunologic activators of the classic complement pathway, such as urate crystals, which may be part of the pathophysiologic process of gout. 

In the alternate pathway, the early complement components (C1, C4, and C2) are bypassed and C3 is activated directly by such things as bacterial endotoxins, cobra venom factor, lipopolysaccharide, and aggregated immunoglobulin (mainly IgA, but also IgE). C3 nephritic factor is an unusual substance capable of activating the alternate complement system within the glomerulus, producing glomerular injury.

Complement assays can be used clinically to help determine the causes and pathomechanisms of certain diseases. For example, activation of the complement cascade can produce local deposition of C3, which can be seen with special histologic techniques. If a patient has widespread activation of the complement system, then serum assays of C3 levels might be decreased. In particular, activation of the classic complement pathway decreases levels of the early complement components, namely C1, C4, and C2. 

In contrast, activation of the alternate complement pathway, which bypasses these early complement components, decreases levels of C3, but the levels of the early factors (C2 and C4) are normal.


36. A 19-year-old female is being evaluated for recurrent facial edema, especially around her lips. She also has recurrent bouts of intense abdominal pain and cramps, sometimes associated with vomiting. Laboratory examination finds decreased C4, while levels of C3, decay-accelerating factor, and IgE are within normal limits. These findings are most likely to be associated with a deficiency of

    a. β2-integrins

    b. C1 esterase inhibitor

    c. Decay-accelerating factor

    d. Complement components C3 and C5

    e. NADPH oxidase


ANS: B

Note: Deficiencies of components of the complement system are associated with specific abnormalities.

Patients with congenital deficiencies in the early components of the complement cascade have recurrent symptoms resembling those of systemic lupus erythematosus due to the deposition of immune complexes.

Patients with deficiencies of the middle complement components (C3 and C5) are at risk for recurrent pyogenic infections, while those lacking terminal complement components (C6, C7, or C8, but not C9) are prone to developing recurrent infections with Neisseria species. 

A deficiency of decay-accelerating factor (DAF), which breaks down the C3 convertase complex, is seen in paroxysmal nocturnal hemoglobinuria (PNH), a disorder that is characterized by recurrent episodes of hemolysis of red cells because of the excessive intravascular activation of complement. 

Deficiencies of C1 esterase inhibitor result in recurrent angioedema, which refers to episodic nonpitting edema of soft tissue, such as the face. Severe abdominal pain and cramps, occasionally accompanied by vomiting, may be caused by edema of the gastrointestinal tract. To understand how a deficiency of C1 inhibitor can cause vascularly produced edema (angioedema), note that not only does C1 inhibitor inactivate C1, but it also inhibits other pathways, such as the conversion of prekallikrein to kallikrein and kininogen to bradykinin. 

A deficiency of C1 inhibitor also leads to excess production of C2, a product of C2 called C2 kinin, and bradykinin. It is the uncontrolled activation of bradykinin that produces the angioedema, as bradykinin increases vascular permeability, stimulates smooth muscle contraction, dilates blood vessels, and causes pain. 

In contrast, a defect involving β2-integrins is seen with leukocyte adhesion deficiency, while defects involving NADPH of leukocytes are characteristic of chronic granulomatous disease.


37. Which one of the listed substances is produced by the action of lipoxygenase on arachidonic acid, is a potent chemotactic factor for neutrophils, and causes aggregation and adhesion of leukocytes?

    a. C5a

    b. Prostacyclin

    c. IL-8

    d. Thromboxane A2

    e. Leukotriene B4


ANS: E

Note:

Products of arachidonic acid (AA) metabolism are involved extensively in inflammation. In this pathway, arachidonic acid is broken down into leukotrienes (vasoconstrictors) and prostaglandins (vasodilators). Arachidonic acid is a polyunsaturated fatty acid that is normally found esterified in plasma membrane phospholipids. It is released by the activation of phospholipases, such as phospholipase A2. Cyclooxygenase transforms AA into the prostaglandin endoperoxide PGG2, which is then converted into PGH2 and subsequently into three products: thromboxane A2 (TxA2), prostacyclin (PGI2), and the more stable prostaglandins PGE2, PGF2, and PGD2. Thromboxane, found in platelets, is a potent platelet aggregator and blood vessel constrictor.

In contrast, prostacyclin, which is found in the walls of blood vessels, is a potent inhibitor of platelet aggregation and is also a vasodilator. Prostaglandin E and prostacyclin probably account for most of the vasodilation that is seen in inflammation. The prostaglandins are also involved in producing pain and fever in inflammation. In contrast to cyclooxygenase, lipoxygenase converts AA into hydroperoxyl derivatives, namely 12-HPETE in platelets and 15-HPETE in leukocytes. 5-HPETE gives rise to HETE and the leukotrienes (Lts). While many substances can be chemotactic, few are known to be as potent as several of the leukotrienes. Leukotriene B4 is a potent chemotactic agent that also causes aggregation and adhesion of leukocytes. Additionally, leukotrienes C4, D4, and E4 cause increased vascular permeability, bronchoconstriction, and vasoconstriction. Other chemotactic factors for neutrophils include C5a and IL-8, but these substances are not formed from arachidonic acid.


38. During acute inflammation, histamine-induced increased vascular permeability causes the formation of exudates (inflammatory edema). Which one of the listed cell types is the most likely source of the histamine that causes the increased vascular permeability?

    a. Endothelial cells

    b. Fibroblasts

    c. Lymphocytes

    d. Mast cells

    e. Neutrophils


ANS: D

Note: Vasoactive amines are important mediators of the early signs and symptoms of acute inflammation. Two important vasoactive amines are histamine and serotonin. 

Histamine is found in mast cells, basophils, and platelets, and is primarily responsible for the initial swelling found in acute inflammation. This swelling results from histamine binding to H1 receptors and increasing the permeability of venules. Histamine release is induced by temperature changes (both hot and cold), antibodies (a type I hypersensitivity reaction), anaphylatoxins, IL-1, and IL-8. Neuropeptides, such as substance P, can cause vasodilation and increased vascular permeability directly and by stimulating histamine release by mast cells. 

Serotonin (5-hydroxytryptamine) is found in platelets and enterochromaffin cells and has actions similar to those of histamine, although these may not be physiologically significant in humans.


39. What type of leukocyte actively participates in acute inflammatory processes and contains myeloperoxidase within its primary (azurophilic) granules and alkaline phosphatase in its secondary (specific) granules?

    a. Neutrophils

    b. Eosinophils

    c. Monocytes

    d. Lymphocytes

    e. Plasma cells


ANS: A

Note: There are several types of leukocytes; all have specific structures that enable them to  participate in specific types of inflammatory reactions. Acute inflammatory processes, such as pyogenic bacterial infections and tissue necrosis, are associated with infiltrates of neutrophils into tissue and increased numbers of neutrophils in the blood; hence neutrophils are thought of as acute inflammatory cells. 

Neutrophils are also called polymorphonuclear leukocytes (PMNs or “polys”) because they characteristically have nuclei with three to five lobes. Myeloperoxidase is an enzyme within the primary (azurophilic) granules of neutrophils, while alkaline phosphatase is an enzyme in their secondary (specific) granules. Neutrophils have a short life span and do not divide. IL-1 causes an increased number of neutrophils to be released from the bone marrow. In contrast, chronic inflammatory processes are associated with increased numbers of monocytes and lymphocytes. 

Monocytes are mononuclear leukocytes with a “bean-shaped” or “horseshoe-shaped” nucleus. Their tissue form is called a macrophage or histiocyte. The activated form of macrophages have abundant eosinophilic cytoplasm and are called epithelioid cells. They secrete many different types of products and may fuse to form giant cells. 

Lymphocytes are smaller mononuclear leukocytes that have a round to oval nucleus and little cytoplasm. There are two types of lymphocytes, B lymphocytes and T lymphocytes. These types of lymphocytes look histologically identical. B lymphocytes (B cells) mature into plasma cells, which have an eccentric nucleus with a “clock-face” appearance of their chromatin. Plasma cells secrete immunoglobulin, while certain T lymphocytes (T cells) secrete lymphokines. Numbers of lymphocytes are increased in acute viral infections or chronic disease.

Eosinophils are bilobed leukocytes that contain abundant eosinophilic granules within their cytoplasm. These granules contain many different types of substances, such as major basic protein (which is toxic to helminthic parasites), arylsulfatase (which neutralizes leukotrienes), and histaminase (which neutralizes histamine). They participate in specific types of inflammatory processes, such as allergic disorders, parasitic infections, and some diseases of the skin. 

Basophils are a type of leukocyte that have numerous deeply basophilic granules within their cytoplasm that completely hide the nucleus. Basophils participate in certain specific types of immune reactions because they have surface receptors for IgE. When activated they release vasoactive substances, such as histamine. Mast cells, although not exactly the same as basophils, are found in tissue and are very similar to basophils.


40. Histologic sections of lung tissue from a 68-year-old female with congestive heart failure and progressive breathing problems reveal numerous hemosiderin-laden cells within the alveoli. Theses “heart failure cells” originate from alveolar

    a. Endothelial cells

    b. Eosinophils

    c. Lymphocytes

    d. Macrophages

    e. Pneumocytes


ANS: D

Note: 

Most tissue macrophages originate from a committed bone marrow stem cell that differentiates into a monoblast and then a promonocyte, and then finally matures into a monocyte in the circulating peripheral blood. When called upon, the circulating monocyte can enter into an organ or tissue bed as a tissue macrophage (sometimes called a histiocyte). Examples of tissue macrophages are Kupffer cells (liver), alveolar macrophages (lung), osteoclasts (bone), Langerhans cells (skin), microglial cells (central nervous system), and possibly the dendritic immunocytes of the dermis, spleen, and lymph nodes. The entire system, including the peripheral blood monocytes, constitutes the mononuclear phagocyte system. In the lung, alveolar macrophages can phagocytize the red blood cells that accumulate in alveoli in individuals with congestive heart failure. These cells contain hemosiderin and are referred to as “heart failure cells.”


41. By definition, granulomas are composed of 

    a. Cholesterol clefts

    b. Collagen

    c. Endothelial cells and fibroblasts

    d. Epithelioid cells

    e. Hemosiderin-laden macrophages


ANS: D

Note: 

Granulomatous inflammation is characterized by the presence of granulomas, which by definition are aggregates of activated macrophages (epithelioid cells). These cells may be surrounded by mononuclear cells, mainly lymphocytes, and multinucleated giant cells, which result from the fusion of several epithelioid cells together, may be present. Granulomatous inflammation is a type of chronic inflammation initiated by a variety of infectious and noninfectious agents. Indigestible organisms or particles, or T cell–mediated immunity to the inciting agent, or both, appear essential for formation of granulomas. Although tuberculosis is the classic infectious granulomatous disease, several other infectious disorders are characterized by formation of granulomas, including deep fungal infections (coccidioidomycosis and histoplasmosis), schistosomiasis, syphilis, brucellosis, lymphogranuloma venereum, and cat-scratch disease. In sarcoidosis, a disease of unknown cause, the granulomas are noncaseating, which may assist in histologic differentiation from tuberculosis. No organisms are found in the noncaseating granulomas of sarcoidosis.


42. A 47-year-old male presents with pain in the midportion of his chest. The pain is associated with eating and swallowing food. Endoscopic examination reveals an ulcerated area in the lower portion of his esophagus. Histologic sections of tissue taken from this area reveal an ulceration of the esophageal mucosa that is filled with blood, fibrin, proliferating blood vessels, and proliferating fibroblasts. Mitoses are easily found, and most of the cells have prominent nucleoli. Which one of the following correctly describes this ulcerated area?

    a. Caseating granulomatous inflammation

    b. Dysplastic epithelium

    c. Granulation tissue

    d. Squamous cell carcinoma

    e. Noncaseating granulomatous inflammation


ANS: C

Note: 

Tissue repair occurs through the regeneration of damaged cells and the replacement of tissue by connective tissue. Tissue repair involves the formation of granulation tissue, which histologically is characterized by a combination of proliferating fibroblasts and proliferating blood vessels. Proliferating cells are cells that are rapidly dividing and usually have prominent nucleoli. This histologic feature should not be taken as a sign of dysplasia or malignancy. It is important not to confuse the term granulation tissue with the similarsounding term granuloma. The latter refers to a special type of inflammation that is characterized by the presence of activated macrophages (epithelioid cells).


43. A routine H&E histologic section from an irregular white area within the anterior wall of the heart of a 71-year-old male who died secondary to ischemic heart disease reveals the myocytes to be replaced by diffuse red material. This material stains blue with a trichrome stain. Which one of the listed statements correctly describes this material?

    a. It is secreted by endothelial cells and links macromolecules to integrins

    b. It is secreted by fibroblasts and has a high content of glycine and hydroxyproline

    c. It is secreted by hepatocytes and is mainly responsible for intravascular oncotic pressure

    d. It is secreted by monocytes and contains a core protein that is linked to mucopolysaccharides

    e. It is secreted by plasma cells and is important in mediating humoral immunity


ANS: B

Note: The extracellular matrix (ECM) is composed of fibrous structural proteins and interstitial matrix, the latter being composed of adhesive glycoproteins embedded within a ground substance. The structural proteins of the ECM include collagen fibers, reticular fibers, and elastic fibers. 

Collagen is a triple helix of three polypeptide α chains that is secreted by fibroblasts and has a high content of glycine and hydroxyproline. Collagens may be either fibrillar or nonfibrillar. The fibrillar (interstitial) types of collagen (types I, III, and V) are found within the ECM (interstitial tissue), while the nonfibrillar type IV collagen is found within the basement membranes, which are special organizations of the interstitial matrix found around epithelial, endothelial, and smooth-muscle cells. Type I collagen is found in skin, tendon, bone, dentin, and fascia; type II collagen is found only in cartilage; and type III collagen (reticulin) appears in the skin, blood vessels, uterus, and embryonic dermis.

The adhesive glycoproteins include fibronectin and laminin. Laminin, the most abundant glycoprotein in basement membranes, is a cross-shaped glycoprotein that is capable of binding multiple matrix components, such as type IV collagen and heparan sulfate. It also binds to specific receptors on the surface of some cells. Fibronectin, secreted by fibroblasts, monocytes, and endothelial cells, is also capable of binding many substances, such as collagen, fibrin, proteoglycans, and integrins. Basically, fibronectin links extracellular matrix component and macromolecules to integrins and is chemotactic for fibroblasts and endothelial cells. Instead of being crossshaped like laminin, fibronectin is a large glycoprotein composed of two chains held together by disulfide bonds. Albumin is secreted by hepatocytes and is mainly responsible for intravascular oncotic pressure, while immunoglobulins are secreted by plasma cells and are important in mediating humoral immunity.


44. A 27-year-old female presents because of trouble with her vision. Physical examination reveals a very tall, thin female with long, thin fingers. Examining her eyes reveals the lens of her left eye to be in the anterior chamber. Her blood levels of methionine and cystathionine are within normal levels. This patient’s signs and symptoms are primarily due to 

    a. Abnormal copper metabolism

    b. Decreased levels of vitamin D

    c. Decreased lysyl hydroxylation of collagen

    d. Defective synthesis of fibrillin

    e. Defective synthesis of type I collagen


ANS: D

Note:

Several diseases result from abnormalities involving defects in structural proteins. Marfan’s syndrome is an autosomal dominant disorder that results from defective synthesis of fibrillin causing connective tissue abnormalities. It is characterized by specific changes involving the skeleton, the eyes, and the cardiovascular system. Skeletal changes seen in individuals with Marfan’s syndrome include arachnodactyly (spider fingers) and a large skeleton causing increase in height. Eyes in patients with Marfan’s syndrome typically have a subluxed lens (ectopia lentis) in which the lens is found in the anterior chamber. The lens dislocation in Marfan’s syndrome is usually upward, in contrast to the downward dislocation seen with homocystinuria. Cardiovascular lesions associated with Marfan’s syndrome include MV prolapse and cystic medial necrosis of the aorta.

Ehlers-Danlos syndrome (EDS) refers to a group of related disorders characterized by defects in collagen synthesis or structure. These defects produce abnormalities of the skin and joints. The skin in these patients is fragile and hyperextensible, while the joints are hypermobile. There are at least 10 variants or subtypes of EDS. Type IV EDS is related to abnormal type III collagen, and as such is associated with ruptured intestinal organs and blood vessels. Type IX EDS involves abnormalities of copper metabolism. There is increased copper in cells, but a decreased copper level in the blood. Type VI EDS is characterized by decreased lysyl hydroxylation of collagen, which causes decreased cross-linking of collagen. Only collagen types I and III are affected. Osteogenesis imperfecta (OI) also results from defective synthesis of type I collagen. These patients have “brittle bones” and also typically develop blue scleras and hearing loss. Decreased levels of vitamin D can produce rickets in children or osteomalacia in adults.


45. Which one of the listed changes correctly describes the pathophysiology involved in the production of pulmonary edema in patients with congestive heart failure?

    a. Decreased plasma oncotic pressure

    b. Endothelial damage

    c. Increased hydrostatic pressure

    d. Increased vascular permeability

    e. Lymphatic obstruction


ANS: C

Note:

Edema is the accumulation of excess fluid in the interstitial tissue or body cavities. It may be caused by inflammation (inflammatory edema) or it may be due to abnormalities involving the Starling forces acting at the capillary level (noninflammatory edema or hemodynamic edema). Inflammatory edema is caused by increased capillary permeability, which is the result of vasoactive mediators of acute inflammation. An exudate is inflammatory edema fluid resulting from increased capillary permeability. It is characterized by a high protein content, much cellular debris, and a specific gravity greater than 1.020. Pus is an inflammatory exudate containing numerous leukocytes and cellular debris. In contrast, transudates result either from increased intravascular hydrostatic pressure or from decreased osmotic pressure. They are characterized by a low protein content and a specific gravity of <1.012. Noninflammatory edema is the result of abnormalities of the hemodynamic (Starling) forces acting at the level of the capillaries. Increased hydrostatic pressure may be caused by arteriolar dilation, hypervolemia, or increased venous pressure. Hypervolemia may be caused by sodium retention seen in renal disease, and increased venous hydrostatic pressure can be seen in venous thrombosis, congestive heart failure, or cirrhosis. Decreased plasma oncotic pressure is caused by decreased plasma protein, the majority of which is albumin. Decreased albumin levels may be caused by loss of albumin in the urine, which occurs in the nephrotic syndrome, or by reduced synthesis, which occurs in chronic liver disease. Lymphatic obstruction may be caused by tumors, surgical resection, or infections (for example, infection with filarial worms and consequent elephantiasis).


46. Which one of the listed clinical scenarios best illustrates the concept of active hyperemia?

    a. A 22-year-old second-year medical student who develops a red face after being asked a question during a lecture

    b. A 37-year-old male who develops massive swelling of the scrotum due to infection with Wucheria bancrofti

    c. A 69-year-old male who dies secondary to progressive heart failure and at autopsy is found to have a “nutmeg” liver

    d. A 6-year-old boy who develops the sudden onset of intense scrotal pain due to testicular torsion

    e. A 71-year-old female who develops perifollicular hemorrhages due to a deficiency of vitamin C


ANS: A

Note:

Hyperemia refers to excess amounts of blood within an organ. It may be caused by increased arterial supply (active hyperemia) or impaired venous drainage (passive hyperemia). Examples of active hyperemia include increased blood flow during exercise, blushing (such as embarrassment associated with being asked a question during a lecture), or inflammation. Examples of passive hyperemia, or congestion, include the changes produced by chronic heart failure. These changes include chronic passive congestion of the lung or the liver. The lung changes are characterized by intraalveolar, hemosiderinladen macrophages, called “heart failure cells.” Congestion in the liver is characterized by centrilobular congestion, which is seen grossly as a “nutmeg” appearance of the liver. 

In contrast to hyperemia, hemorrhage refers to the leakage of blood from a blood vessel. Blood may escape into the tissue, producing a hematoma, or it may escape into spaces, producing a hemothorax, hemopericardium, or hemarthrosis. Superficial hemorrhages into the skin or mucosa are classified as petechiae (small, pinpoint capillary hemorrhages), purpura (diffuse, multiple superficial hemorrhages), or ecchymoses (larger, confluent areas of hemorrhages).


47. Procoagulant factors produced by endothelial cells include

    a. Thrombomodulin

    b. Prostacyclin

    c. von Willebrand factor

    d. Thromboxane A2

    e. Fibrinogen


ANS: C

Note:

The three main components of hemostasis include endothelial cells, platelets, and the coagulation system. Endothelial cells exhibit both procoagulant and anticoagulant properties. Their procoagulant activities involve activation of the extrinsic coagulation cascade by their production of tissue factor (thromboplastin) and stimulation of platelet aggregation by their production of von Willebrand factor and platelet-activating factor. Their anticoagulant activities involve the production of prostacyclin (PGI2), thrombomodulin, and plasminogen activator. The contrasting actions of the arachidonic acid metabolites prostacyclin and thromboxane A2 (TxA2) produce a fine-tuned balance for the regulation of clotting. TxA2, a product of the cyclooxygenase pathway of arachidonic acid metabolism, is synthesized in platelets and is a powerful platelet aggregator and vasoconstrictor. The prostaglandin PGI2, also a product of the cyclooxygenase pathway but produced by endothelial cells, inhibits platelet aggregation and causes vasodilation. Aspirin, a cyclooxygenase inhibitor, blocks the synthesis of both TxA2 and PGI2 and is used in the treatment of coronary artery disease. Fibrinogen, which is produced by the liver and not endothelial cells, is cleaved by thrombin to form fibrin.


48. Which one of the listed laboratory findings is most consistent with an individual who is not taking any medication but has a familial deficiency of coagulation factor VII, assuming all other coagulation factors to be within normal limits?

【Pathology question】General Pathology Q48

ANS: A

Note:

The coagulation cascade involves the formation of fibrin through the intrinsic, extrinsic, and common pathways. The intrinsic pathway is initiated by contact of factor XII with several types of biologic surfaces. Activated XII (XIIa) initiates the formation of XIa and IXa. The extrinsic pathway is initiated by contact of tissue factor with factor VII. Activated factor VII acts together with IXa, VIIIa, and platelet factor 3 (PF-3), which is a phospholipid complex located on the surface of platelets, to produce activated factor X. This begins the common pathway, which continues with the interaction of Xa, Va, PF-3, and Ca++ to cleave prothrombin, forming thrombin, which in turn cleaves fibrinogen to form fibrin.

Two laboratory tests that are used to evaluate the functioning of the coagulation cascade are prothrombin time (PT) and partial thromboplastin time (PTT). Abnormalities of the extrinsic pathway prolong (not shorten) the PT, while abnormalities of the intrinsic pathway prolong (not shorten) the PTT. Note that abnormalities of the common pathway prolong both the PT and the PTT. To illustrate, deficiencies of factor VII produce an abnormal (prolonged) PT with a normal PTT. Compare these results to each of the following: a normal PT with an abnormal PTT can be seen with deficiencies of factors XII, XI, IX, or VIII, while abnormal PT and PTT are seen with deficiencies of X, V, prothrombin, or fibrinogen.


49. A postmortem clot is most likely to

    a. Grossly display features of recanalization

    b. Grossly have lines of Zahn

    c. Grossly have the appearance of “chicken fat” overlying “currant jelly”

    d. Microscopically appear attached to the wall of the blood vessel

    e. Microscopically have alternating layers of cells and platelets


ANS: C

Note:

Thrombi may form within the heart, the arteries, or the veins. When formed within the heart or the arteries, thrombi may have laminations, called the lines of Zahn, formed by alternating layers of platelets admixed with fibrin, separated by layers with more cells. Mural thrombi within the heart are associated with myocardial infarcts and arrhythmias, while thrombi in the aorta are associated with atherosclerosis or aneurysmal dilatations. Arterial thrombi are usually occlusive; however, in the larger vessels they are not. Venous thrombi, which are almost invariably occlusive, are found most often in the legs, in superficial varicose veins or deep veins. Those of the larger outflow veins of the leg may embolize. It is important to be able to tell postmortem clots from venous thrombi. The postmortem clot is usually rubbery, gelatinous, and lacks fibrin strands and attachments to the vessel wall. Large postmortem clots may have a “chicken fat” appearance overlying a dark “currant jelly” base.


50. What is the most common site of origin of thrombotic pulmonary emboli?

a. Deep leg veins

b. Lumen of left ventricle

c. Lumen of right ventricle

d. Mesenteric veins

e. Superficial leg veins


ANS: A

Note:

An embolus is a detached intravascular mass that has been carried by the blood to a site other than where it was formed. Emboli basically can be thrombotic or embolic, but most originate from thrombi. These thromboemboli, most of which originate in the deep veins of the lower extremities, may embolize to the lungs. The majority of small pulmonary emboli do no harm, but, if they are large enough, they may occlude the bifurcation of the pulmonary arteries (saddle embolus), causing sudden death. Arterial emboli most commonly originate within the heart on abnormal valves (vegetations) or mural thrombi following myocardial infarctions. If there is a patent foramen ovale, a venous embolus may cross over through the heart to the arterial circulation, producing an arterial (paradoxical) embolus.

Types of nonthrombotic emboli include fat emboli, air emboli, and amniotic fluid emboli. Fat emboli, which result from severe trauma and fractures of long bones, can be fatal as they can damage the endothelial cells and pneumocytes within the lungs. Air emboli are seen in decompression sickness, called caisson disease or the bends, while amniotic fluid emboli are related to the rupture of uterine venous sinuses as a complication of childbirth. Amniotic fluid emboli can also lead to a fatal disease, disseminated intravascular coagulopathy (DIC), which is marked by the combination of intravascular coagulation and hemorrhages. In this setting DIC results from the high thromboplastin activity of amniotic fluid.

Comments

Popular posts from this blog

【方剂学练习题】 各论:第十四章 治风剂

    【方剂学练习题】 各论:第十四章 治风剂 答案 选择题 名词解释 86. 凡以疏散外风或平息内风等作用为主,用于治疗风病的方剂,统称为治风剂。   87. 内风是指由于脏腑功能失调所致的风病,其发病多与肝有关,也称为肝风,有肝风上扰、热盛风动、阴虚风动及血虚生风等。   填空题 是非改错题 简答题 131. 风热或风湿浸淫血脉则伤阴血,苦寒渗利之品亦可伤及阴血,故用当归、生地、胡麻仁以养血活血,滋阴润燥,既补已伤之阴血,且达 “ 治风先治血,血行风自灭 ” 之意,又制约诸药之温燥,皆为佐药。   132. 镇肝熄风汤主治类中风。本方所治之类中风,张氏称之为内中风,系由肝肾阴虚,肝阳偏亢,阳亢化风,气血逆乱所致。风阳上扰,故见头目眩晕、目胀耳鸣、脑部热痛、面红如醉。肝肾阴亏,水不上济,故心中烦热。肝阳上亢,气血逆乱,并走于上,遂致卒中。轻则风中经络,肢体渐觉不利,口眼渐形斜;重则风中脏腑,眩晕颠仆,昏不知人,即《素问 · 调经论》所谓 “ 血之与气,并走于上,则为大厥,厥则暴死。气复反则生,不反则死 ” 。本方证以肝肾阴虚为本,阳亢化风、气血逆乱为标,本虚标实,本缓标急,当急则治标,以镇肝息风为主,佐以滋养肝肾为法。   133. 消风散主治风疹、湿疹。是由风湿或风热之邪侵袭人体,浸淫血脉,内不得疏泄,外不得透达,郁于肌肤腠理之间所致,故见皮肤瘙痒、疹出色红,或抓破后渗溢津水。风胜则痒,痒自风来,故治宜疏风止痒为主,配以除湿、清热、养血之法。舌苔及脉像的表现为苔白或黄,脉浮数。   134. 凡以疏散外风或平息内风等作用为主,用于治疗风病的方剂,统称为治风剂。风病分为外风与内风。外风是指外来风邪,侵袭人体肌表、经络、筋骨、关节等。由于外感六淫常相兼为病,故其证又有风寒、风湿、风热等区别。其他如风邪毒气从皮肤破伤之处侵袭人体而致破伤风等,亦属外风。内风是指由于脏腑功能失调所致的风病,其发病多与肝有关,有肝风上扰、热盛风动、阴虚风动及血虚生风等。外风宜疏散,内风宜平息。因此,本章方剂分为疏散外风剂和平息内风剂两类。   135. 肝为刚脏,喜条达而恶抑郁,过用重镇之品以强制,势必影响其疏泄条达之性,故又以茵陈、川楝子、生麦芽清泄肝热,疏理肝气,以顺肝性,利于肝阳的平降镇潜,均为佐药。此外,生麦芽可防止

【TCM】失眠 insomnia

失眠 失眠是一种临床常见病症,中医称为“不寐”、“不得眠”、“不得卧”等。失眠主要是因为心神不宁,心失所养或心为邪扰所致,从而产生入睡困难、睡眠时间缩短、睡眠质量下降等情况。  失眠将严重影响患者的日常生活与工作,从而对安眠药产生依赖。但是,长期服用安眠药会带来潜在的不良反应,比如情绪激动、白细胞减少、抑郁焦虑、幻觉等。 中医在治疗失眠上有着悠久的历史,早在殷墟甲骨文中便有关于“寐”、“梦”等记载。 《黄帝内经·素问》则最早阐述了病机“胃不和则卧不安”。 此外,中医治疗手法多样,疗效也甚好。 内容: 1. 失眠的病因病机 2. 辨证论治 3. 常用中药      a. 芳香中草药      b. 疏经活络中草药 4. 针灸      三焦针法      基本取穴      心脾两虚型失眠症      痰热扰心型失眠症 5. 资料整理来源 1. 失眠的病因病机 病因:七情内伤,病后体虚,年迈失养,或是肝气不调 病机:郁而化火,导致心神失养,不得安眠 证型: 虚证:心脾两虚 主要证型 忧思伤脾 → 脾不健运,暗耗脾血 →  脾不健运则无以上输精微 → 血虚则无以养心供血 → 心失所养,神失所安 表现:失眠不安,并多伴惊惕易醒,心悸多梦,头晕等症 心脾两虚将亏损心气,同时,肾虚以将导致失眠。 当肾水不足,心火上炎之时,亦将出现心神不宁的虚实夹杂之失眠 实证: i. 痰热扰心 a. 脾胃失运,则易痰浊内扰 b. 因湿热邪气引动 c. 因七情内伤,郁而化火 d. 因积食化热,导致痰热内生 上犯清空,客扰心神,而致不寐 表现:失眠不寐,伴胸闷呕恶,嗳气目眩,口苦 ii. 肝郁化火 七情内伤,郁而不舒 → 肝为抑郁,气机不得条畅 → → 肝郁乘土,耗伤脾血,心神不养而难眠 → 郁扰心神,烦闷扰神不得眠 肝气郁结日久,肝郁化火,邪火扰动心神,心神不安而不寐 iii. 肝阳上亢  多见于高血压患者 肾水不足 → 抑或是阳亢难遏 → 阴阳失衡,阴不制阳,肝阳上亢而不寐 《灵枢·口问》论述了阴阳与睡眠的关系 “阳气尽,阴气盛,则目瞑;阴气尽,阳气盛,则寤矣。 ” 表现:失眠伴有头胀痛,目赤耳鸣,面色潮红,口干口苦,烦躁易怒 更多有关于高血压: 【TCM】中医治疗高血压 (Hypertension) 【Pharmacology】Antihypertensive 2. 辨证论治 虚证:心脾两

【Covid-19】增加免疫力 & 抗炎的中药 (现代研究证实)

内容: 1. 免疫力介绍      a. T 细胞      b. B 细胞      c. NK 细胞      d. 巨噬细胞      e. 细胞因子 2. 免疫调节中药的应用      a. 增强免疫      b. 抑制免疫      c.双向调节免疫 3. 抗炎 4. 整理 5. 参考文献 1. 免疫力 是人体的防御机制,通过识别和消灭外来侵入的任何异物,来达到保护人体的功效。 中医讲究“防未病”,“正气存内,邪不可干”。在某种意义上,正气相当于免疫力。 如今,新冠肺炎肆虐全球,病毒不断变种,是人们措手不及。 此时,我们应注重提高自身的免疫力,以达到防治新冠肺炎的重要作用。 许多中药不仅能“祛邪”,还能“扶正”,帮助提高机体免疫功能以抵御疾病。 中药调节免疫功能不仅能增强免疫,还有些中药可调节过度的免疫反应,缓解炎症风暴所致的损伤。 中药对于机体免疫系统的调节功能与T 细胞、B 细胞、NK 细胞、巨噬细胞以及细胞因子的产生和活性密切相关。 T 细胞 (T lymphocytes): 直接杀死受感染的宿主细胞 激活其他免疫细胞,产生抗体 产生细胞因子 (cytokines),调节免疫反应。 B 细胞 (B lymphocyte): 产生抗体:制造称为抗体的 Y 形蛋白质 Y 形蛋白质可锁定 & 标记入侵细胞 不直接参与消灭:标记让其他免疫细胞破坏 NK 细胞 (natural killer cell): 顾名思义,其功能为杀死异常的细胞 控制感染:通过限制其扩散 & 随后的组织损伤 巨噬细胞(macrophage): 具有强大的识别、吞噬、清除细菌及外来异物的功能 细胞因子 (cytokines): 是一种信号蛋白,调节炎症反应 调节不当,会导致細胞因子風暴 (Cytokine storm) 由各种免疫细胞,包括巨噬细胞、淋巴细胞 2. 免疫调节中药的应用 2a.增强免疫 “正气存内,邪不可干”,而参类可大补元气、补气活血、滋养脏腑之气,因此参类能增强免疫,如: 人参 (Ginseng) & 丹参 (Danshen Root) 人参和丹参提取物:对流感病毒A 感染小鼠具有免疫调节作用,能促进肺特异性抗体IgA 的产生 明党参 (Changium smyrnioides Wolff) 明党参煎液和多糖:显著增加小鼠胸腺指数、脾指数、外

【Pathology question】General Pathology part 2

【Pathology question】General Pathology part 2 DIRECTIONS : Each item below contains a question or incomplete statement followed by suggested responses. Select the one best response to each question. 51. A 9-year-old boy suddenly develops severe testicular pain. He is taken to the emergency room, where he is evaluated and immediately taken to surgery. There his left testis is found to be markedly hemorrhagic due to testicular torsion. This abnormality caused a hemorrhagic testicular infarction because of      a. Arterial occlusion      b. Septic infarction      c. The collateral blood supply of the testis      d. The dual blood supply of the testis      e. Venous occlusion ANS: E Note: Infarcts are localized areas of ischemic coagulative necrosis.  They can be classified on the basis of their color into either red or white infarcts, or by the presence or absence of bacterial contamination into either septic or bland infarcts. White infarcts : pale or anemic infarcts, are usually the res

【Covid-19】9个具有抗病毒的中药

抗病毒药物(antiviral drug) 是一类用于特异性治疗病毒感染的药物。但抗病毒药物非万能药,其原因为它无杀死病毒的功能,而是通过干扰病毒复制周期的某个环节,实现抵抗病毒的进一步感染。 病毒复制周期: 吸附 ⏩ 侵入 ⏩ 脱壳 ⏩ 病毒大分子的合成 (核酸 & 蛋白质) ⏩ 病毒的装配与释放 抗病毒药物的功效: 直接抑制或杀灭病毒 干扰病毒吸附 阻止病毒穿入细胞 抑制病毒生物合成 抑制病毒释放 增强宿主抗病毒能力 与抗生素的差别? 抗病毒药物:抑制病毒的发展 抗生素:消灭细菌 与杀病毒剂(viricide)的差别? 抗病毒药物:抑制体内的病毒 杀病毒剂:消灭体外的病毒 以下内容皆从 余亚茹, 鲁鹏飞, 王红霞, & 黄宝康. (2020, May 1).  中药防治新型冠状病毒肺炎概述 . 药学实践杂志. http://yxsj.smmu.edu.cn/cn/article/doi/10.12206/j.issn.1006-0111.202003098. 所整理出来的 9个具有抗病毒的中药 1. 金银花 Honeysuckle Flower  功效:清热解毒,疏散风热 适用于:风热感冒,温病发热 有效成分:有机酸类、黄酮类、三萜皂苷类、挥发油类 绿原酸 、咖啡酸、槲皮素、木樨草苷: 抗流感病毒的活性成分 抑制甲型流感病毒、病毒性心肌炎、疱疹病毒及腺病毒 绿原酸、咖啡酸 :与流感病毒被膜上的神经氨酸酶结合, 抑制流感病毒的早期复制 金银花多糖: 减轻肺病变程度、增强机体免疫力 其降低甲型流感病毒FM1 株滴鼻感染小鼠的死亡率,延长小鼠的生存时间,减轻肺病变程度,抗病毒 植物miRNA-miR2911 新型的抗流感病毒成分 结构稳定、耐煎煮,在体内肺部富集 直接靶向H1N1、H5N1 和H7N9 甲型流感病毒 抑制H1N1 编码的PB2 和NS1 蛋白的表达 → 抑制病毒复制   miR2911: 抑制H5N1 和H7N9 甲型流感病毒 (在体内、体外均可) 2. 麻黄 Herbs Ephedrae 功效:发汗解表 适用:风寒感冒,胸闷喘咳 有效成分:生物碱类成分 甲基麻黄碱、L-麻黄碱 & D-伪麻黄碱 抑制甲型流感病毒的 体外增殖 麻黄碱:抑制H1N1 型流感病毒对 犬肾细胞 的感染,并呈浓度依赖性。 3. 黄岑 Baikal Sk

【Pharmacology】CHP 1 General Principles

CHP 1 General Principles DIRECTIONS: Each item below contains a question or incomplete statement followed by suggested responses. Select the one best response to each question. 1. Of the many types of data plots that are used to help explain the pharmacodynamics of drugs, which plot is very useful for determining the total number of receptors and the affinity of a drug for those receptors in a tissue or membrane?      a. Graded dose-response curve      b. Quantal dose-response curve      c. Scatchard plot      d. Double-reciprocal plot      e. Michaelis-Menten plot ANS: C Note:  Based on the concept that, for most situations, the association of a drug with its receptor is reversible, the following reaction applies: where D is the concentration of free drug, R is the concentration of receptors, DR is the concentration of drug bound to its receptors, and KD (equal to k2/k1) is the equilibrium dissociation constant. The affinity of a drug for its receptor is estimated from the dissociatio

【Acupoints】治疗消渴病 / 糖尿病的穴位

治疗糖尿病的穴位 关于糖尿病: 【TCM】糖尿病 / 消渴症 Diabetes mellitus 【Pharmacology】Drug for diabetes i. 体针 上消(肺热津伤) 肺俞、 脾俞 、 胰俞 、尺泽、 曲池 、廉泉、承浆、 足三里 、 三阴交 *烦渴、口干 +金津、玉液 中消(胃热炽盛) 脾俞 、胃俞、 胰俞 、 足三里 、 三阴交 、内庭、 中脘 、阴陵泉、 曲池 、 合谷 *大便秘结 + 天枢 、支沟 下消(肾阴亏虚) 肾俞 、 关元 、 三阴交 、 太溪 *视物模糊 +太冲、光明 阴阳两虚 气海 、 关元 、 肾俞 、命门、 三阴交 、 太溪 、复溜 ii. 按摩 肥胖或超重患者:腹部按摩 中脘 、水分、 气海 、 关元 、 天枢 、水道等 点穴减肥: 合谷 、内关、 足三里 、 三阴交 按顺序: 1. 肺俞 【位置】位于第三胸椎棘突下,后正中线旁开1.5寸, 属膀胱经。 【主治】咳嗽,气喘,吐血,骨蒸,潮热,盗汗,鼻塞。 【糖尿病应用】上消(肺热津伤) 2. 脾俞 【位置】在脊柱区,第11胸椎棘突下,后正中线旁开1.5寸,属膀胱经。 【主治】 消化系统疾病,例如腹胀,腹痛,胃痛,呕吐,泄泻,黄疸,急、慢性胃炎,胃痉挛,胃下垂,神经性呕吐,肝炎,细菌性痢疾。 肾炎,贫血,糖尿病,水肿,进行性肌营养不良。 血证,例如吐血,便血,尿血,慢性出血性疾患。 【糖尿病应用】上消(肺热津伤)、中消(胃热炽盛) 3. 胰俞 【位置】在第8胸椎棘突下旁开1.5寸,膈下穴与肝俞穴 【主治】胃脘痛,呃逆,口苦咽干,大便不调,多饮多尿,盗汗遗精,痛痛厌食,酸楚楚 【糖尿病应用】上消(肺热津伤)、中消(胃热炽盛) 4. 尺泽 【位置】让患者采用正坐、仰掌并微曲肘的取穴姿势,尺泽穴位于人体的手臂肘部,取穴时先将手臂上举,在手臂内侧中央处有粗腱,腱的外侧外即是此穴(或在肘横纹中,肱二头肌桡侧凹陷处)。 【主治】主治咳嗽,气喘,咯血、胸部烦满,咽喉肿痛,肘臂挛痛等。 【糖尿病应用】上消(肺热津伤) 5. 曲池 【位置】屈肘成直角,当肘弯横纹尽头处;屈肘,于尺泽与肱骨外上髁连线的中点处取穴。 【主治】手臂痹痛、上肢不遂、热病、高血压、癫狂;腹痛、吐泻、咽喉肿痛、齿痛、目赤肿痛,瘾疹、湿疹、瘰疬等 【糖尿病应用】上消(肺热津伤) 6. 廉泉 【位置】人体的颈部,当前正中

【方剂学练习题】 各论:第十七章 祛痰剂

  【方剂学练习题】 各论:第十七章 祛痰剂 答案: 选择题 名词解释 66. 凡以消除痰涎作用为主,用于治疗各种痰病的方剂,统称为祛痰剂。   是非改错题 填空题 简答题 93. 凡以消除痰涎作用为主,用于治疗各种痰病的方剂,统称为祛痰剂。属于 “ 八法 ” 中 “ 消法 ” 的范畴。痰随气而升降流行,气滞则痰聚,气顺则痰消。诚如庞安常所言: “ 善治痰者,不治痰而治气,气顺则一身津液亦随气而顺矣。 ” 故祛痰剂中常配伍理气药。至于痰流经络、肌腠而为瘰疬、痰核者,又常结合软坚散结之品,随其虚实寒热而调之。   94. 滚痰丸主治实热老痰证。治宜泻火逐痰。方中礞石味甘咸而性平质重,咸能软坚,质重沉坠,下气坠痰以攻逐陈积伏匿之顽痰,并平肝镇惊而治痰火上攻之惊痫,且制以火硝,《本草问答》谓: “ 礞石,必用火硝煅过,性始能发,乃能坠痰,不煅则石质不化,药性不发。又毒不散,故必煅用。 ” 煅后攻逐下行之力尤强,为治顽痰之要药,故以之为君。   95. 二陈汤主治湿痰证。方中用半夏、橘红等药燥湿化痰,恐辛温香燥耗肺伤阴,故配伍少许乌梅收敛肺气,与半夏相伍,散中有收,使祛痰而不伤正,且有 “ 欲劫之而先聚之 ” 之意,为佐药。   96. 半夏白术天麻汤主治风痰上扰证。症见眩晕,头痛,胸膈痞闷,恶心呕吐,舌苔白腻,脉弦滑。其功效为化痰息风,健脾祛湿。本方乃二陈汤去乌梅,加天麻、白术、大枣而成。方中半夏辛温而燥,燥湿化痰,降逆止呕;天麻甘平而润,入肝经,善于平肝息风而止眩晕。二者配伍,长于化痰息风, “ 头旋眼花,非天麻、半夏不除 ” ,共为君药。白术健脾燥湿; 茯苓健脾渗湿,以治生痰之本,与半夏、天麻配伍,加强化痰息风之效,共为臣药。橘红理气化 痰,使气顺痰消,为佐药。使以甘草调药和中,煎加姜、枣以调和脾 胃。诸药合用,共奏化痰息风、健脾祛湿之效。   97. 祛痰剂中每多配伍健脾祛湿之药,有时酌配益肾之品,以杜绝生痰之本,而图标本同治,因痰饮常由湿聚而成,而湿主要源之于脾。正如《医宗必读》中说:“脾为生痰之源,治痰不理脾胃,非其治也。”强调治痰当健脾,脾复健运之常,而痰自化矣。然痰饮与肾亦有密切关系,如肾不至水,则水泛为痰,故《景岳全书》中说:“五脏之病,虽俱能生痰,然无不由乎脾肾。”张介宾并指出“善治痰者,惟能使之不生,方是补夭之手”。   98.

【TCM】高跟鞋女性如何养生

高跟鞋女性如何养生 内容: 为什么职业女性都要穿上高跟鞋? 优美的代价。。。 如何保养及足部按摩? 参考文献 为什么职业女性都要穿上高跟鞋? 1. 更有女性气质 有研究表明,当女性穿高跟鞋时,步伐会更小更快,走路时膝盖和髋部的弯曲程度都较小,而胯骨的旋转和倾斜程度增大。”也就说明,女性穿高跟鞋时走路更女性化,因此可以让她们看上去更有女性气质。 2. 更有活力 穿上高跟鞋除了能使人视觉上高一些,也能使人背部自然的呈弧形,产生一个挺胸抬头的作用。如今使女性看起来更优雅及有活力。 中医方面也对此有所解释。穿高跟鞋时,脚尖点地支撑身躯的重量,在脚尖中又以足大趾为主要着力点,而足大趾是足厥阴肝经的起点,足厥阴肝经的作用是调动肾中所藏之精气上达于心,并外输于四肢百骸,使人体处于一种生机勃发的状态。因此,穿上高跟鞋的女性就处于让肝经兴奋、精气外泄的状态,使人看起来有生气、有活力。 优美的代价。。。 1. 容易诱发趾外翻、趾囊炎、锤状趾、趾骨头缺血性坏死等疾患 因为现代职业女性工作压力大、工时长,长时间穿高跟鞋,导致身体重心前移,全身 60% 的重量需要前脚掌支撑,使足尖负重增大。 如果穿上前部是尖的高跟鞋,女性得将类似于方形的足前部挤进锥形的窄小的鞋尖内,则更使双足备受折磨。如此,女性将容易有足部的疾病,如诱发趾外翻、趾囊炎、锤状趾、趾骨头缺血性坏死等疾患。 2. 腰痛 如同前面所述,穿上高跟鞋,能使人背部呈现弧形。但穿久,则导致过度的腰部后伸,背肌收缩绷紧,腰椎小关节和关节囊处于紧张状态。从而造成关节囊和腰背肌就会发生劳损,引起腰痛。 3. 月经不调 因足厥阴肝经一直处于兴奋的状态,导致肝火上逆、肾中精气外泄过度,阴虚而火旺。 《灵枢·经脉第十》曰:“肝足厥阴之脉,起于大趾丛毛之际,上循足跗上廉,去内踝一寸,上踝八寸,交出太阴之后,上腘内廉,循股阴,入毛中, 过阴器,抵小腹 ,挟胃,属肝,络胆,上贯膈,布胁肋,循喉咙之后,上入颃颡,连目系,上出额,与督脉会于巅;其支者,从目系下颊里,环唇内;其支者,复从肝,别贯膈,上注肺。” 足厥阴肝经循行的部位包含二阴、小腹。因此,肝火旺盛会导致月经不调,如月经提前、延后或过少等病变。 4. 乳房胀痛,乳腺增生 职业女性压力大,情志不遂时,则导致足厥阴肝经气“上贯膈,布胁肋”之时,造成肝气不舒,郁结而成乳房胀痛,乳腺增生。 5. 口干咽痒 肾